Of Particular Significance

Did the LHC Just Rule Out String Theory?!

POSTED BY Matt Strassler

POSTED BY Matt Strassler

ON 09/17/2013

Over the weekend, someone said to me, breathlessly, that they’d read that “Results from the Large Hadron Collider [LHC] have blown string theory out of the water.”

Good Heavens! I replied. Who fed you that line of rubbish?!

Well, I’m not sure how this silliness got started, but it’s completely wrong. Just in case some of you or your friends have heard the same thing, let me explain why it’s wrong.

First, a distinction — one that is rarely made, especially by the more rabid bloggers, both those who are string lovers and those that are string haters. [Both types mystify me.] String theory has several applications, and you need to keep them straight. Let me mention two.

  1. Application number 1: this is the one you’ve heard about. String theory is a candidate (and only a candidate) for a “theory of everything” — a silly term, if you ask me, for what it really means is “a theory of all of nature’s particles, forces and space-time”. It’s not a theory of genetics or a theory of cooking or a theory of how to write a good blog post. But it’s still a pretty cool thing. This is the theory (i.e. a set of consistent equations and methods that describes relativistic quantum strings) that’s supposed to explain quantum gravity and all of particle physics, and if it succeeded, that would be fantastic.
  2. Application number 2: String theory can serve as a tool. You can use its mathematics, and/or the physical insights that you can gain by thinking about and calculating how strings behave, to solve or partially solve problems in other subjects. (Here’s an example.) These subjects include quantum field theory and advanced mathematics, and if you work in these areas, you may really not care much about application number 1. Even if application number 1 were ruled out by data, we’d still continue to use string theory as a tool. Consider this: if you grew up learning that a hammer was a religious idol to be worshipped, and later you decided you didn’t believe that anymore, would you throw out all your hammers? No. They’re still useful even if you don’t worship them.

BUT: today we are talking about Application Number 1: string theory as a candidate theory of all particles, etc.

Now what’s so silly about this notion that the LHC has ruled out string theory is that the whole reason a lot of people hate string theory is that it doesn’t make any testable predictions! So obviously you can’t rule it out with current experiments… that would require testable predictions!

Caution! Do not interpret the above bold-faced statement to mean that string theory as a theory of all particles etc. makes no predictions at all. String theory certainly makes predictions. For instance (taking string theory in its most vanilla form), string theory predicts that all particles are actually tiny strings.  And if you were to take one electron with a motion-energy of a million million million GeV or more, and take another electron with similar energy but moving in the opposite direction, and slam them into each other, you would find the scattering is governed by the formulas for how relativistic quantum strings bounce off each other.  Specifically,

  • The probability that they would scatter at large angle is extremely small.
  • The probability that they scatter at a small angle has a bell-shaped distribution centered around zero angle; the width of the bell shrinks as the energy increases.
  • The overall scattering rate increases with energy.
  • And since all particles (not just electrons) are strings, if string theory is right, the same results should hold for scattering of any two particles, not just electrons.

These features are very different from what you’d expect if particles had no detectable size or shape.  Also, the formulas are more precise than my words, so these are things you could check in detail, not just qualitatively.

There’s only one problem: although these predictions would allow definitive test of the theory, the tests can’t be carried out with current or currently imaginable technology.  In current experiments, the scattering energy is far too low to detect whether the particles of nature are tiny strings, as small as (vanilla) string theory would suggest.  The pattern of scattering we see when two electrons scatter is what you’d expect if particles had no intrinsic size or shape. In short, we can’t actually build a particle accelerator capable of testing these predictions; our technology is about a thousand million million times below where it would need to be, so this isn’t going to happen soon. Nor does nature provide a natural laboratory; cosmic rays aren’t nearly energetic enough.   The problem is practical, not one of principle.

So why would anyone even think for a moment that string theory was ruled out by the LHC? It apparently comes from the following line of argument.

  • String theory predicts supersymmetry.
  • Supersymmetry has been ruled out by the LHCb experiment at the LHC, so string theory is ruled out too.

Both of these statements are wrong, in multiple ways.

First, does string theory predict supersymmetry? Even if supersymmetry is present in nature, there is nothing in string theory that predicts that supersymmetry lies hidden at the energies that can be accessed by the LHC; i.e., even if there exist superpartner particles, they may be far, far too heavy for the LHC to find.  Furthermore, although supersymmetry is in some subtle ways built deeply into string theory (as we currently understand it), that doesn’t mean that it shows up in a simple way: it is not necessarily the case that there will be a recognizable superpartner particle for every particle, as traditional supersymmetry predicts.

The only situation in which we expect supersymmetry to show up at the LHC is if supersymmetry is solving the naturalness problem of the Standard Model, and thus stabilizing the hierarchy between the extreme weakness of gravity and the strengths of the other forces.  For short, let’s call this “natural supersymmetry”.  If supersymmetry exists in nature but has nothing to do with the naturalness problem, we don’t have any reason to expect to find any sign of it at the LHC.

Second, did LHCb rule out supersymmetry? Well, I just told you that’s impossible, because if supersymmetry exists but natural supersymmetry does not, then superpartners could be far, far, far out of reach of the LHC. But did LHCb rule out natural supersymmetry? No; I’ve said this twice before, and I’ll just send you to the links (here and here). What has actually come close to ruling out natural supersymmetry (but we’re not quite there yet) are the results of the searches for superpartner particles done at ATLAS and CMS. LHCb is just trying to steal their thunder, but their claims in the press are bombastic at best and inaccurate at worst.

Still, even if LHCb, ATLAS and CMS together ruled out natural supersymmetry definitively (and that won’t happen til 2016, most likely), that would not rule out supersymmetry (which might exist but not have anything to do with the naturalness problem of the Standard Model) or string theory (which might not even exhibit supersymmetry in any usual way).

So forget it. The LHC will not rule out string theory; it won’t even come close. The machine is powerful, but not even remotely powerful enough to do this.  It’s like asking an ameoba to climb Mount Everest.

[Note Added for experts regarding making predictions in string theory: In response to complaints by commenter Peter Woit (see discussion below in the comments) let me reiterate the importance of my remark that I was taking string theory in its vanilla form.  Keep in mind that quantum field theory in general doesn’t predict how protons scatter or whether they even exist — you have to choose a specific quantum field theory, or at least a specific class of such theories, before you can do that.  Similarly, string theory in general doesn’t predict how particles scatter.  But if you choose a specific vacuum of string theory, or a specific class of such vacua, then you can make predictions.  (Not that we understand string theory well enough yet to understand the different vacua in detail — another practical problem.)  The `vanilla form’ of string theory is a class of vacua in which the particles of nature are tiny strings that have no very strong forces acting on them.  Tell me we’re in such a vacuum: then I can make predictions.  In other vacua, I’d make other predictions.  In any vacuum, predictions can be made, though the answers depend, to a greater or lesser degree, on which vacuum you are in.  Presumably, even if string theory is right, only an interplay between experiment and theory can tell us which vacuum we might be in, just as such an interplay was necessary to tell us which specific quantum field theory to use in particle physics.]

Share via:

Twitter
Facebook
LinkedIn
Reddit

147 Responses

  1. I believe what you said was actually very logical. However,
    think on this, suppose you composed a catchier title?
    I ain’t saying your information isn’t solid, however suppose you added something that makes people
    desire more? I mean Did the LHC Just Rule Out String Theory?!
    | Of Particular Significance is a little boring.
    You should glance at Yahoo’s front page and see how they create article titles to get people
    interested. You might add a video or a picture or two to get people excited about everything’ve written. Just my opinion, it would bring your
    posts a little bit more interesting.

  2. I believe that is one of the so much important information for me.

    And i am happy reading your article. However should observation on few general
    things, The site style is ideal, the articles is in point of fact great : D.
    Excellent activity, cheers

  3. I am now not certain wwhere you aree getting your info, but gooid topic.

    I needs to spend a while fnding out ore or working out more.
    Thank you for wonderful info I used to be in search of tis info for my mission.

  4. It’s the best time to make some plans for the future and it’s time to be happy.
    I have read this post and if I could I wish to suggest
    you few interesting things or suggestions. Perhaps you can write next articles referring to this article.
    I desire to read more things about it!

  5. a mr muller commented on another (non particle physics)forum

    ‘Note that this is a rather unimaginative attidude. It is very possible that a string theory would make predictions about things that are already tested, but appear in current theories as arbitrary parameters. E.g. like predicting a realtion between the mass of the proton, neutron and electron. Reduction of the number of arbitrary parameters can be a very good reason to accept a theory.

    For comparison: Nuclear physics, in particular the theory that nuclei are built out of protons and neutrons, predicts it should be possible to smash nuclei apart, of fuse them. But even in a civilization with a technology that would not be able to do that, or foresee they would ever be able to do that (e.g. because they had not discovered electricity), such a theory would have a great appeal. As it would explain the atomic weights (as determined by purely chemical methods) of nearly 100 elements in terms of just proton mass, neutron mass and some interaction term based on their number, to a quite high precision. While otherwise these would just be 92 arbitrary ‘constants of nature’.”

    please could you respond

    thank you

    duncan

    1. The commenter doesn’t know enough particle physics to make an entirely sensible comment… some of it makes sense, some of it doesn’t. That makes it hard to respond to. What’s your question, really?

  6. First, I think we all should appreciate the time invested by Dr. Strassler in clarifying his views and the current state of the art. Let’s take for a given that String Theory in “application 2”, i.e. as a valuable tool for studying very complex models that may or may not be applicable to the real world, is established and extremely useful.
    Still, most of us pedestrians (and blog-readers) are more interested in the case for “application 2”. The lack of evidence in LHC for “natural” SUSY, and the difficulty in identifying an istantiation of String Theory that can claim to be applicable, even in principle, to our phenomenological world, is something that has turned some of us from very interested in ST to skeptic about this specific application of the ST efforts.
    If we are not to consider it a solution to the “naturalness problem” of the Standard Model, I think it is very hard to build a compelling case for SUSY against Okham’s razor. And if we do not have a case for SUSY, it looks also very hard to make a case for ST as a “theory of everything”, throwing in unobserved extra-dimensions, landscapes, etc. This is of course the (or a) layman’s perspective, which is generally only interested in explaining this universe, not in postulating a possible quasi-infinite set of unobservable universes, nor in fictitious spaces that are actually mathematical devices.

    Again, thanks for the time, the effort and the insightful information.

    1. For decades we have been trying to come to terms are particles lumps or waves? It is going to take probably centuries before man has enough basic information to comprehend quantum theory as a complete theory. In the meantime sustain other ideas which may not have any relevance to the ontological world. String theory suggests that which we do not know is made up of super infinitesimal fibres. I suppose therefore these are unable of demonstrating this duality? And become solid again being smaller than a sub quantum particulate?

      In a horse race there can only be one winner, if they all start the race with a common number 1 and as soon as the winning horse nose touches the finish line all the others have their number changed to a zero. All the losing horses become a zero but continue to participate in the competition even though this Information has determined that it is finished.

      Nature is Information and does not require particles, dynamics or any form of physics – and can change everything in the cosmological arena and how we understand it.

  7. Dear Prof. Strassler,

    I would like to listen to your oppinion on http://arxiv.org/abs/1106.3548, where ST seems to emerge from usual Topological Field Theory in the Pure Spinor Formalism. Do you think the derivation is correct? What implications should this have for Application (1) and (2) of ST you mentioned in this blog entry?
    Many Thanks

    Num

  8. WOW~! what a comment page! Thanks to both DRs for letting their hair down and giving us mortals a glimpse of the sordid underbelly of HEP!!!

    I guess, as a fan of both blogs.. the question is.. does a #1 style string theory require SUSY? and if so does it requiire it at a level we should have seen at the LHC? and so if then no SUSY is found (as in LHCb) is #1 string theory proven false.

    1. As Matt Strassler pointed out in his comments, Woit has nothing to do with HEP anymore. So this isn’t “the … underbelly of HEP”.

      1. Prof S clearly said that something Prof W said about HEP cost him millions in funding.. maybe “inner workings”or “catfights n a sack” would be better than “sordid underbelly” but I went with that…

  9. Not falsifiable – it seems this is the rule of the physics community and dogma so why cannot it be applied to String Theory? How does it get away with it? I assume dangerously is because it presents an new idea which has not been presented before? So it must be investigated just to check it out.

  10. This is such a lively debate. Had the main protagonists signed a tv deal to air it live I am sure Prof. Strasslar would have raised his much needed research funding.However I cant help suspecting that when things become heated they both or one of them goes into ‘stealth’ mode and assume a different nome de plume or pseudoname and begins to wield the mentioned hammer on the unsuspecting participant!

  11. This discussion is sorta misleading, because the mainstream physics (including the string theorists itself) don’t understand, what this theory does at the conceptual level, so they don’t recognize the confirmation of string theory in the LHC results.

    For example, the string theorists predicted, that the existence of extradimensions would stabilize the microscopic black holes – and it did really happen at LHC, as these microblack holes were prepared routinely in form of new atom nuclei even at RHIC.

    So that the string theory failed quantitatively – but conceptually I don’t see any problem with its postulates. These postulates are just inconsistent mutually, which prohibits the string theory in predictability and practical testability.

    1. “the string theorists predicted, that the existence of extradimensions would stabilize the microscopic black holes – and it did really happen at LHC, as these microblack holes were prepared routinely in form of new atom nuclei even at RHIC.”

      You are profoundly mixed up. I’m too tired to set you straight today, but I do want to assure other readers that they should ignore this.

      1. “the string theorists predicted, that the existence of extradimensions would stabilize the microscopic black holes – and it did really happen at LHC, as these microblack holes were prepared routinely in form of new atom nuclei even at RHIC.” This is very interesting. I would like to know more about this.

  12. Dear Matt, I am sure glad to see you go through this effort here.

    There is so much confusion out there in the lay (and, what is worse, not-so-lay) public about the nature of string theory, and so woefully litte (close to none) public discussion that is both informed and balanced. I am glad to see you go through the trouble, I do know that it costs time and energy, but it is very much necessary for the de-confusion of the interested public and also for the mental health of the physics community at large, it seems.

    In all modesty, maybe I may point out that in reaction to a steady flow of questions similar to the one that you are reacting to here, a while back I had started to write, on the “nLab”-wiki site, a “string theory FAQ” that is at least intended to provide educated and balanced information, the link is here:

    http://ncatlab.org/nlab/show/string+theory+FAQ

    (Of course if anyone comes and finds the education and/or balancing of the information there wanting, I promise to listen to constructive criticism and try to improve the discussion further. And generally, all educated input to this nLab page and the nLab in general from anyone interested is welcome).

    Looking through your discussion here, I think there is a good bit of overlap and agreement between the points you highlight and the discussion on that FAQ page.

    1. Thank you for this FAQ, it is a great resource! I learned a lot (relatively speaking) about string theory today. I can’t believe I missed nLab so far.

    1. Huh. I missed that paper completely, and I haven’t heard anyone else mention it (which is a bad sign). It’s a very implausible suggestion, but made by a very experienced and well-regarded scientist. I will have to read it and get back to you.

    2. On first reading: it’s a clever proposal, but (as you would expect) it involves several subtle points, any one of which could invalidate the proposal. I will have to think about this carefully — as carefully as Bekenstein has done — before I could comment intelligently about it.

  13. To Matt Strassler: “in fact you only get electromagnetism after the Higgs mechanicsm mixes the weak isospin force and hypercharge forces together, so in a sense electromagnetism isn’t even the fundamental thing you should be trying to explain. What’s my point? It turned out that asking “why is the strength of electromagnetism 1/137?” was a bad question.” This sounds that Maxwell’s equations or the Standard Model are insufficient to understand “electromagnetism” or behaviors of “a photon” at more (or the most) fundamental level. I hear some people say(pretend?) as if almost everything about electromagnetism/light/photon is understood but this is not the case, it seems. There are more fundamental layers to electromagnetism/photon. Is this the case? My question:are there other ways to detect the effect of more fundamental constituents(or mechanisms) of particles/fields/vacuum without smashing particles at currently unattainable extremely high energy range? Perhaps by precision measurement? (for example, is it possible to detect/extract quantum fluctuation (or other properties) of vacuum by observing a photon?) or some other subtle detection methods?? If you are aware of these candidates, could you list some of them with a bit of explanations?

    1. The point is simple enough: we understand everything about electromagnetism EXCEPT

      1) the overall strength of the electric force

      2) the mass of the electron, the muon, and the other charged particles

      These things cannot be computed *within* the theory of electromagnetism, and in fact are clearly not related in a simple way to the more fundamental aspects of nature that somehow determine them.

      But tell me these two classes of things, and then I can calculate everything about photons and electrons and muons and protons and what they do (electromagnetically) in the world.

      1. If I understand correctly what you’re saying here then I disagree since QED can only be renormalized within perturbation theory and, therefore,
        strictly makes sense only as an effective theory.

      2. To Matt Strassler,

        How can you calculate/explain the followings?:
        0)spin seems quantized so, I guess it might be some kind of wave. Is spin wave? if so what kind of wave? Is it a standing wave or not? What does your calculation say about the behavior of “spin wave?”?
        1)how “spin” arises with its observed value -> why is photon’s spin always 1? and electron 1/2?
        2)why spin direction (axis) of any? massless particle is always found along its traveling direction while massive one can point to any direction (its spin direction unrelated to its traveling direction?)? (does this mean a photon and gluon spin axis is “completely” aligned along its traveling direction without any deviation or uncertainty?) Why so? Why only massless particle has coupling with its traveling direction but not the massive one?
        3)what is the size(spread, volume) (of the momentum of) a photon? Does the spread have a shape? Does a photon have a sharp edge/boundary?
        4)How is a photon moving/behaving(and looks like) at very small scale? For example, how is a photon affected by quantum fluctuation in vacuum (at around? planck distance?)? what does the calculation say about this? Or, how does a photon behave inside a black hole? Is a photon(or something, some kind of wave?) still moving at the bottom of the black hole? Nothing can move at the point(singularity?)?? or the concept of a mathematical point is nonsense and does not exist in correct physics or in nature? or something else exists at very small scale and its structure/property etc. allow movement (motion of wave? vibration)? I guess if black hole has some heat and entropy, something must be moving(=cannot stop moving?) inside (at the bottom?). Anyway, what happens to a photon falling into a black hole around and inside the event horizon and finally at the bottom?
        5)Do two photons collide or interact with each other? If so how? How can a photon interact with anything(like an electron) if its time is not ticking? If photon’s time is not ticking how can its waves vibrate?
        6)A photon has no thickness along its traveling direction? I guess it must at least look that way conceptually because things moving at the speed of light (appear to) have no thickness along its traveling direction. Is this the case? And a photon(electromagnetic fields) seems (and is) not vibrating(cannot vibrate?) toward its traveling direction (=photon is always transverse wave and never longitudinal wave? if so why?). I heard reference frame of a photon is not permitted in relativity but from a photon’s own view, is itself vibrating or not? And, how does a photon see another photon traveling toward the same direction next to each other? Does a photon say the other photon is vibrating with zero speed (=standing wave?)?

        1. Hi – it’s not going to work very well to ask so many questions at once… Not only don’t I have time to answer them, but the questions layer misunderstandings upon each other in such a way that I can’t easily pull them apart and answer you simply.

          If you know a little bit of math (i.e. a little calculus) and a little physics (what you would learn in your first semester of undergraduate physics class), you could try to start with http://profmattstrassler.com/articles-and-posts/particle-physics-basics/fields-and-their-particles-with-math/

          1. To Matt Strassler,

            Can you at least answer 0), 1) and 2) about spin? I do not remember any of your article or the SM explains how spin (of photon, electron etc.) emerges. Spin is treated as an intrinsic property of particles without explanations of how it emerges within currently established physics. Is this not the case?

        2. It may sound strange at first, but indeed the quantization of spin *can* naturally be explained in terms of wavefunctions, namely in terms of wavefunctions on a phase space which is a 2-sphere. Kirillov and others have amplified this in textbooks on geometric quantization and on the orbit method, but probably this nice fact of mathematical physics hasn’t percolated much into the quantum-public perception. Details are spelled out here:

          http://ncatlab.org/nlab/show/geometric+quantization+of+the+2-sphere

  14. Lovely, a reiteration of the string wars from the mid 2ks, more or less verbatim. Same silly arguments that had identically zero effect in academia, but strangely seems to capture the hearts and mind of netizens.

    1. Wrong, on two counts.

      1) I am making a very different argument from that made by the string devotees, a group I do not belong to.

      2) The string wars of that era (and before) had a significant impact in academia. The demographics of departments and people’s careers (including mine) were affected.

      1. Matt you are making perfectly sensible arguments, its just that you are making the same arguments all of us gave Peter the first time around.

        Id argue that departamental hiring trends didn’t change as a result of the string wars, but rather due to the advent of the LHC. Do you know any senior physicist who changed his mind due to having read two popular books and reading a few blogs, because I don’t.

        Anyway, I’ll bow out now.

  15. Prof. Strassler,

    You say “I agree that string theory predicts nothing about particle physics.”

    Many would agree that there is constant progress in using String Theory as a computational tool. But since we know that the Standard Model works so well and yet there are so many open challenges associated with it, why is the research attempting to explain the underpinnings of the Standard Model not given the highest priority? Don’t we run the risk of placing excessive emphasis on BSM physics, while ignoring the growing number of unsettled issues facing high-energy theory?

    1. On the one hand, if you remember this article

      http://profmattstrassler.com/2012/08/15/from-string-theory-to-the-large-hadron-collider/

      that is exactly what some people are doing. Learning how to calculate more efficiently and effectively in the Standard Model is one of the highest-priority items on the agenda.

      But your second question reflects some confusion. “Don’t we run the risk of placing excessive emphasis on Beyond-the Standard Model (BSM) physics, while ignoring the growing number of unsettled issues facing high-energy theory?” We already know that many and perhaps all of the unsettled issues CANNOT be settled within the Standard Model. (The reason we know this is that our understanding of quantum field theory is now very good, as far as we can tell, and we know, from experience playing around with different quantum field theories, when a question has a chance of being answered internally to a theory and when input from outside is needed.) That’s precisely why so many people work on BSM physics… it’s because they want to settle issues within the Standard Model, and they know they can only do it by working outside the model.

      One can however ask whether working on the subtle points of string theory contributes to this effort. The answer depends. I see no sign that Application Number 1 of string theory will contribute. However, Application Number 2 does and may continue to contribute. In that limited sense, string theory *does* contribute to particle physics — and I misspoke by not making clear that my remark about string theory *not* contributing related to Application Number 1.

      1. Let me rephrase my second question in a different way.

        What evidence there is that BSM physics is at the root of the mass and flavor hierarchy problems, or of the naturalness problem, or of the anomalous magnetic moment problem, to pick few examples? Is failure to find acceptable answers through the “the beaten path” a solid criterion to give up searching for solutions within SM? To be specific, what if we currently fail to appreciate all the ramifications of Wilson’s Renormalization Group or of the nonlinear dynamics of quantum fields?

        1. Hmm. I would say the evidence is overwhelming. But to explain that non-technically… that seems difficult. I have never thought about how to argue this to a non-expert. It really delves into how much we know about field theory, where our confidence derives from, what experiment tells us… I don’t think you can argue this in a few paragraphs.

          But maybe I’m just too tired right now to think of a clever way to do it.

  16. Whilst I admit to being initially baffled by the comparison between string theory and a mythical hammer, the prominence of Mjollnir as a fertility symbol does, at the very least, give a new perspective on the importance of extended objects.

  17. Matt, related to the issue of advances in string theory allowing things previously out of reach be calculated, and problems to be solved that were intractable before, is there anything you could explain about this new “amplituhedron” method discussed at https://www.simonsfoundation.org/quanta/20130917-a-jewel-at-the-heart-of-quantum-physics ? The article at quanta magazine is much better than most layman-targeted articles on similar topics–it’s quite breathless and the “artistic rendition” of the new geometric object is laughable, but it does try to include some details and it quotes some people who I imagine know what they’re talking about, to the effect that this represents an incredible improvement over previous calculation methods. But I can’t figure out if the geometrical object they have found underlying twistor representations, the amplituhedron, has actually given rise yet to the possibilities of new twistor calculations, or if it’s just a sounder mathematical foundation for what was already possible with twistor diagrams.

    1. Importantly, notice that the first half of the article — which is about the advances that have actually occurred so far — have nothing to do with string theory or even gravity. There’s been no progress on gravity — yet.

      I think there is a danger that some of this advance will prove to be over-hyped; as you say, there is some breathlessness here, among the experts as well as the journalist. But what they have been able to do — the first half of the article — has been very impressive. So I would say I am cautiously optimistic that this mathematical advance (which seems real) will have real implications, even for practical calculations in particle physics, or at least for our understanding of field theory, somewhere down the line. And I would also be very optimistic that they will make some progress in quantum gravity calculations. Be careful, however: the journalist has mixed things up a bit, and you should avoid being misled. The type of quantum gravity calculations required are not reliant on string theory or its details. Indeed these advances are not likely to cast any light on string theory itself, just on the quantum gravity part of string theory (or any other candidate theory of everything), and only on the part which has to do with scattering particles off each other via gravitational forces. There will be no insights into black holes, or cosmology, or other tricky problems of quantum gravity, from this line of research.

  18. @ Dilaton,

    Stop your dirty trolling and let this debate follow its due course. It is a very important exchange of ideas and many people here simply do not care about your personal feelings.

  19. Without angular momentum there is no difference between vacuum solution and topological defect in spacetime.
    In quantum physics the distinction between boson and fermion concepts is not clear cut).
    Einstein’s remorse was in Paul Dirac’s evolutionary attempts to reconcile general relativity with quantum mechanics – his resolute faith in the logic of mathematics as a means to physical reasoning (Compactification), his explanation of spin as a consequence of the union of quantum mechanics and relativity, and the eventual discovery of virtual particles. The Dirac equation is consistent with both the principles of quantum mechanics and the theory of special relativity.

    1. @veeramohan
      *Today*, we know that the dark mass and the dark energy are the essential parts of gravity. Newton did not know them. But, his equation is still good for dark mass as it is a mass anyway. It is the same for the General Relativity which deals only about mass and mass-energy. Is dark energy mass-energy? If it is not, GR is of course incomplete. Without knowing exactly what the dark energy is and is about, the gravity cannot be understood. Without knowing what the gravity is, any talking about *Super Unified Theory* is simply *nonsense*.

      For the mainstream physics, the dark energy was discovered only 15 years ago. It is still a mystery for the mainstream physics. The dark energy survey has now begin last week in Chile. Yet, there is no clue on it theoretically in the mainstream physics. One outside the mainstream idea is that the bright light of the omnipotent of the SR has *blinded* views for the tiny gate which enters into the huge territory of the dark energy.

      1. Thank you Mr tienzengong ,
        so SR ought to prevent the mathematical evolution and Compactification along with it – like photos are prevented other than 2 degree of freedom, to remain massless – eventhough they have spin (polarization).
        The extra degree of freedom “c” = c^2 of massive particles, which is rest mass is prevented (blinded) from falling into the gateway towards dark energy ?

  20. Iam sorry, mistakes…. it should be…
    ….But.. extra dimensions
    …. to interpret naturalness into unnaturalness..

  21. Enlightening Professor,
    Mr Tienzen (Jeh-Tween) Gong, Iam not familiar with maths. But eytra dimension was a mathematical evolution at the absence of concrete spacetime – need Compactification for consistancies (mathematical logics) -in which the human understanding of predictions exists.

    /2. Einstein is remorseful for SR having some sort of misleading karma./-
    This is what I can understand, Mr Einstein try to interpret unnaturalness into naturalness and unsatisfied like everyone ?

  22. Apple Boy: Hi, G-grandmother. Professor Matt Strassler had a new post on string-theory. It truly gives a perfectly detailed description about the string theory. But, most importantly, he laid out the issue of SUSY by categorizing it into three groups.
    1. Natural supersymmetry (related to the Standard Model unnatural problem) — testable at LHC.
    2. Supersymmetry proper (not related to Standard Model or anything else) — could exist at very high energy places, far, far, far out of reach of the LHC.
    3. Supersymmetry of string theory — that doesn’t mean that it shows up in a simple way: it is not necessarily the case that there will be a recognizable superpartner particle for every particle, as traditional supersymmetry predicts.
    I have two questions for you.
    a. Is there any supersymmetry in Nature-master’s file box?
    b. How can people on Earth rule out the case 2 and 3 if they are wrong? Per Matt’s saying, we have no way to rule them out if they are simply wrong.

    G-grandmother: This morning, I did peek into Nature-master’s file box. I did see a *supersymmetry* folder, but I cannot find anything about SUSY (with s-particle) or anything about the *supersymmetry of M-string*. But, the answer for your second question can also be the answer for the first one. There is no need to rule them out. As soon as the correct answer is showed to them, they will simply *trash* the others, not wasting one more second on them regardless of whether they are right or wrong. They call it the Occam’s razor, you know.

    Apple Boy: So, all is hinged on the right answer. But, again two questions.
    i. Does anyone (besides the Nature-master himself) know about the right answer?
    ii. How can he convince others about his answer being the right one?

    G-grandmother: Good questions. But, it is not real that big deal. The first step is pointing out the wrong answers, the weeding out, you know. Using string theory as an example, it should have two missions.
    1. Making contacts with the knowing physics, that is, reproducing the known SM particles, especially the 48 matter particles.
    2. Understanding the old mystery. Gravity was kind of mystery. By *fully* understanding the gravity, it will open up the gate for the *Super unification*.
    If string theory does not have the two missions above, it will be no value of any kind as a physics theory. Yet, we all know that it failed on both missions. As we discussed before, the *dark energy* plays 2/3 parts in gravity. Yet, before the discovery in 1998 that the universe’s expansion is accelerating, string theory had no slightest clue about it. All the arguments about whether the string theory has any great *predictions* or not is not very important, as it is already a *failed* theory in terms of the two missions above.

    Apple Boy: So, all is hinged on the dark energy now. A five year plan for the Dark Energy Survey was lunched last week. It is an international collaboration using the Blanco telescope in Chile to study the effects of dark energy on the evolution of the universe through a variety of probes — supernovae, baryon acoustic oscillations, weak gravitational lensing, and counts of galaxy clusters. But, G-grandmother, you have hinted repeatedly that the dark energy is not a mystery but is a piece of cake. Can you elaborate on that a bit more?

    G-grandmother: The dark energy itself is not a mystery, but the story around it was a bit entangled. You know, Einstein came upstairs not too long ago. I put up a welcome-wagon party just for him. We are now good neighbors and friends. We often had good party together. But, one time, I talked to him about the dark energy after I learned it from your brother, and he began to weep. What is the matter, Mr. E, I said. Remorse, remorse, he replied. Oh, Mr. E, Nature-master gave everyone a timer when we went to Earth. When the time up, we have to come back, dropping all the unfinished work behind. You have done so much for the mankind with your SR (special relativity) and GR (general relativity). It is truly not a bit problem for your not knowing the dark energy. But, he bawled even harder and said, “Misled, misled”. Misled of what? I asked. We all know that GR is not complete but still very thankful. Yet, he kept weeping. Is anything about SR, I asked. He nodded. Is an upcoming experiment going to invalidating the SR somewhat? No, he murmured. Then, what is the problem? I asked. “Misled, misled” he murmured. Misled of what? I asked again. He turned in and did not want to talk about anymore.

    Apple Boy: Wow, a truly weird story, entangled indeed. But, we can get three points from this story.
    1. Einstein is confident about the SR to stand all experimental tests.
    2. Einstein is remorseful for SR having some sort of misleading karma.
    3. The misleading of SR is related to the dark energy.
    Am I right? G-grandmother. Can you talk about the dark energy now?

    G-grandmother: Your analysis is the only one that I can came up with too. Now, after thinking about Mr. E’s weeping, I am exhausted today.

      1. Apple Boy: G-grandmother, I think that we had been a bit off-topic in my last comment.

        G-grandmother: Oh, Matt’s topic was about “LHC cannot rule out the string theory”. One of the comments can be that “it (M-theory) can be ruled out by other ways”. But, if without giving some reasons for the comment, it will simply be a hot air comment. By giving a bit supporting reasons, it indeed got a tad too lengthy.

        Apple Boy: Matt is the best physics blogger today. I think that we should just hush up.

        G-grandmother: Physics is very important to the well-being of mankind. Matt has done a great service for people. When you have any question, he is the one who you can ask. If you have any wrong idea, he is the one who has the knowledge to right your wrong. Of course, you must respect his wish, not running your question or comment too long.

  23. Matt—I agree with everything you say, but there might be one way to actually rule out string theory. If the LHC discovers 10 extra dimensions, then I don’t think any 13+1 dimensional string theories are consistent. Of course, we’d have a lot more to think about if that happened 🙂

    1. Well, while this might be true, it would still be very tricky to use the LHC *alone* to convince oneself that whatever one was observing was really the signal of a whole bunch of extra dimensions. It won’t be possible to immediately interpret such a signal in a unique way. So I’m sure the debate would continue for a decade or two.

      1. Very interesting debate between Prof. Strassler and Dr. Woit from which rest of us are learning a lot. As a complete outsider, I wonder if I can make some comments. First, your example of solar system was excellent. Yes, Newton could not predict location of planets but he predicted relation between orbits and periods. That convinced people about Newton’s law of gravitation. Perhaps string theorists have to do something like this. Unfortunately (!!!) since GR and SM came before ST they cannot be claimed as successes for ST. Otherwise they would have been regarded as predictions of ST. OK to assume some vacuum , but predict some observable. If accelerator experiments are out of question in our lifetime, perhaps CMBR details or some such thing I do not know about. Are there ST predictions about CMBR anisotropy which could be observed in near future assuming some initial conditions? BTW excuse me ,I did not know which reply button to press in this long debate!

        1. @k.v.
          ”Yes, Newton could not predict location of planets but he predicted relation between orbits and periods.”

          Uh, that would be Kepler.

          From Wikipedia, the free encyclopedia:

          ”In astronomy, Kepler’s laws of planetary motion are three scientific laws describing orbital motion, originally formulated to describe the motion of planets around the Sun.

          Kepler’s laws are:

          1.The orbit of every planet is an ellipse with the Sun at one of the two foci.
          2.A line joining a planet and the Sun sweeps out equal areas during equal intervals of time.[1]
          3.The square of the orbital period of a planet is proportional to the cube of the semi-major axis of its orbit.
          1.The orbit of every planet is an ellipse with the Sun at one of the two foci.
          2.A line joining a planet and the Sun sweeps out equal areas during equal intervals of time.[1
          3.The square of the orbital period of a planet is proportional to the cube of the semi-major axis of itmobrien2@hotmail.cas orbit.

          1. Dear Professor Strassler,

            Here is a simple question should you care to answer it: A photon in a straight trajectory and I am perpendicular to it: Tell me please how can I see this photon if it is not moving towards me and not incident on anything to prove its existence? Is it transmitting light spherically- therefore I can give witness to its motion?

  24. Quote from S. Weinberg (talking about the making of the Standard Model and describing below the theoretical situation in the 50s):
    There began a period of disillusionment with quantum field theory. The community of theoretical physicists tended to split into what at the time were sometimes called, by analogy with atomic wave functions, radial and azimuthal physicists.
    (ref :http://arxiv.org/abs/hep-ph/0401010)

    A personal epistemological hypothesis for the recent past (80s to 2010s):
    – radial physicists = susy model-building theorists concerned mainly with dynamics at very high (grand unification?) energy and secondly obsessed by the naturalness problem.
    – azimuthal physicists = theorists that would try first to gain a deeper understanding of the quantum mechanism of electro-weak symmetry breaking and then to deepen the relation between spacetime symmetries, gauge principle and its quantum handmaiden : anomaly freedom…

  25. Caution! Do not interpret the above bold-faced statement to mean that string theory as a theory of all particles etc. makes no predictions at all. String theory certainly makes predictions. For instance (taking string theory in its most vanilla form), string theory predicts that all particles are actually tiny strings. And if you were to take one electron with a motion-energy of a million million million GeV or more, and take another electron with similar energy but moving in the opposite direction, and slam them into each other, you would find the scattering is governed by the formulas for how relativistic quantum strings bounce off each other. Specifically,

    The probability that they would scatter at large angle is extremely small.
    The probability that they scatter at a small angle has a bell-shaped distribution centered around zero angle; the width of the bell shrinks as the energy increases.
    The overall scattering rate increases with energy.
    And since all particles (not just electrons) are strings, if string theory is right, the same results should hold for scattering of any two particles, not just electrons.
    These features are very different from what you’d expect if particles had no detectable size or shape. Also, the formulas are more precise than my words, so these are things you could check in detail, not just qualitatively.

    Matt, in regard to your two paragraphs above I have questions:

    Could the formula (derived from observations?*) for colliding particles at relativistic speeds against each other have the same bell-shaped distribution if particles are suspected to be spherical, not stringy? If yes, then why should string theory be needed? If no, then is it because string theory allows for too many possible distribution possibilities to be useful at confining the predictions?

    Why should anyone think that particles may not have a detectable size? A size may not be exactly determinable due to quantum uncertainties but a ball-park idea of size should be acceptable I would think. I feel that something that is zero in size is equivalent to nothing and is not interactable.

    * If it isn’t too much, could you provide a link to a paper that shows the distribution patterns of colliding particles and plots the data in bell curves?

    Thank you,

    Vincent Sauvé

    1. Elementary particles cannot have a formula like the string formula.

      Composite particles like protons and other hadrons CAN have a formula like this; in fact, as far as small-angle distribution, then DO. And that’s why people thought for a while that protons and other hadrons are strings. (And also why now we can view them, to a limited degree, as strings in four spatial dimensions, with one of the dimensions curved, forming an Anti-de-Sitter-like space.) So it will not be immediate that stringy behavior will be uniquely understandable as saying particles are elementary strings… that will take time and investigation, just as all good things do.

      I didn’t say we have any reason to think particles don’t have a detectable size. I only said that until you detect that size, you expect certain features in the scattering, which we observe in our current scattering data. In quantum field theory of elementary particles, such as quantum electrodynamics, the electrons and photons have no intrinsic size. As a result, all scattering amplitudes are approximately scale invariant, and rates for processes such as electron + positron –> muon + antimuon fall as 1/E^2, where E is the scattering energy (with a small correction due to the slowly varying strength of electromagnetism.) If you see a deviation from this behavior (and others), that tells you that the objects that you are scattering off of have a size that you are starting to detect, of order h c / E, where h is Planck’s constant and c is the universal speed limit (i.e. speed of light.)

      There isn’t any paper that answers your question; there are dozens of papers that show lots of data but no one would ask the question the way you asked it. Field theory textbooks will show you formulas for electron + electron –> electron + electron and for electron + positron –> muon + antimuon or photon + photon and many other things; string theory textbooks will show you the Veneziano and Virasoro-Shapiro amplitudes for simple string scattering and some of them will even tell you how to interpret them in terms of scattering angles, though not very many give you plots. Maybe someday I’ll write an article explaining this stuff, but I have no time for it now.

      1. Matt:

        Before people came up with the ideas of string theory there were people smashing particles together and plotting the scatter distributions. Correct? I’m no fan of string theory and don’t care that a string formula appeared to explain the distribution patterns, if that’s what happened. Even if it did purport to explain those patterns that would not make it correct, just one of many ways something might be explained. If something in nature is mysterious its not good to invent something as, or more, mysterious to try to explain it. In a nutshell, that’s my take on string theory.

        What I was trying to get at is, could the distribution pattern observed for colliding particles at relativistic speeds have the same bell-shaped distribution if particles are suspected to be spherical, not stringy? You seem to be saying that Hadrons have a distribution pattern that string theory has a formula for. I wanted to know if an interpretation of observed particle-particle collision has to be by way of a stringy theory? You say there is a difference with elementary particles so let’s skip that for the moment. It does appear that you are saying that these distributions can be interpreted without the string interpretation. Is this correct?

        1. “If something in nature is mysterious its not good to invent something as, or more, mysterious to try to explain it. In a nutshell, that’s my take on string theory.”

          I believe that’s not a good way to think about things. Once upon a time, relativity was very mysterious; so was quantum field theory. But these are not mysterious now, and they were right. If you flee from mysterious things, that probably means you will flee from the truth.

          About scattering: I think you’re confused. But whenever you and I discuss things, I can’t understand what you’re saying, and vice versa. So perhaps we’re suffering, yet again, from mis-communication.

          The history of electrons: electrons have always satisfied point-particle scattering formulas, in which no size is observable.

          The history of protons: protons initially satisfied point-particle scattering formulas, but eventually these failed. From this failure alone, one could not determine shape, only rough size.

          Only with many different experiments could one determine something about shape — but really, that’s the wrong way to say it. Scattering is sensitive not only to shape but to internal structure — is the object soft or hard? does it have hard parts in its interior? What we learned from scattering high-energy electrons off protons is that protons are soft objects, very roughly spherical, with a size of about 10^(-15) meters, with hard point-like objects inside them.

          However, before these things were learned, we first learned that when protons scatter off each other at moderate energy, they behave like strings — to a degree. This is not magic or mysticism — this is the statement that a fast-spinning proton deforms into a long blob, which looks like a fat string. At least, that is the current interpretation of what it means. The fact that these excited states of the neutron and proton lie on this line

          http://inspirehep.net/record/1203862/files/n1.png

          is one of the most striking lines of evidence. This is not the only place you see this; pions and rho mesons behave the same way:

          http://inspirehep.net/record/1235541/files/regge_N.png

          So yes, you should not think of string theory as some mysterious thing: it is a theory of strings, and it would describe any stringy object, including a fast-spinning hadron like a proton, pion or rho meson.

          In short, hadrons are approximately like strings in some particular circumstances.

          But string theory in its modern guise as a candidate “theory of everything” is the idea that particles like electrons and quarks are exactly strings in all circumstances. That’s much more constraining.

          However, these strings (in vanilla string theory) are a million million million times smaller than the proton. Consequently, electrons, quarks, photons, etc will all behave like point-particles, with no size or shape, until we do experiments that reach distance scales of 10^(-33) meters and energies of 10^18 GeV.

          Clearly, no one has, or will perhaps ever, observe that electrons and quarks scatter in such a way to reveal whether they are or are not strings of this incredibly small size. So when you ask me about past experiments — they’re irrelevant. No one has ever or will in your lifetime check whether an electron is a (vanilla) string.

          1. In regard to this part of your reply:

            (Quoting me:)
            Matt: Before people came up with the ideas of string theory there were people smashing particles together and plotting the scatter distributions. Correct? I’m no fan of string theory and don’t care that a string formula appeared to explain the distribution patterns, if that’s what happened. Even if it did purport to explain those patterns […]

            “If something in nature is mysterious its not good to invent something as, or more, mysterious to try to explain it. In a nutshell, that’s my take on string theory.”

            (You wrote:)
            “I believe that’s not a good way to think about things. Once upon a time, relativity was very mysterious; so was quantum field theory. But these are not mysterious now, and they were right. If you flee from mysterious things, that probably means you will flee from the truth.”

            Matt: I don’t know where you get the idea that I or someone else who might agree with my point would be fleeing from the truth if something is mysterious. Where does that come from? I only said that “If something in nature is mysterious its not good to invent something as, or more, mysterious to try to explain it.” I said nothing about fleeing scientific investigation. Sometimes we just have to say that we don’t know why yet.

            I will say that I totally disagree with the hyper-dimensional approach of string theory if that is the right way to refer to it. I’m certainly no expert in such things. But the idea fails the smell test in my mind. Those ideas seem to me to not be parsimonious particularly when it is postulated that these strings are a million million million times smaller than a proton–thus very unlikely to be proved. That resembles the position of a religious person who says that his god exists and can do many things but He is invisible and won’t reveal himself. Such a position means no one can prove it right or wrong but the burden is on the advocate, in any case.

            Regarding relativity theory and other ideas that we take for granted now, most people still don’t have a thorough understanding of these things. A very small percentage of the population has a real good understanding in a few areas related to their schooling or career, but even the most knowledgable of people will be very ignorant in other areas. Yet most people would not try to invent a dimensional model that exceeds the three directional dimensions to that we know of to explain the inner workings of a smartphone or a jet engine or some other item of modern technology. I never flee from the “truth.” Let’s keep in mind that science in the past thought they had the “truth” about many things. And it turned out that many of those “truths” weren’t true after all.

            I appreciate the rest of your explanation in the same posting.

          2. To Matt Strassler,
            You said:
            “The history of protons: protons initially satisfied point-particle scattering formulas, but eventually these failed. From this failure alone, one could not determine shape, only rough size.”

            So, even observing “muonic hydrogen” only helps determine proton size more accurately but not determine its shape? If so, how/why? If even more heavy charged particle instead of (than) muon is used to form an atom, does it reveal proton shape? If you keep increasing the particle’s mass, does it eventually reveal the proton shape?

      2. “”” string theory textbooks will show you the Veneziano and Virasoro-Shapiro amplitudes for simple string scattering and some of them will even tell you how to interpret them in terms of scattering angles, though not very many give you plots. Maybe someday I’ll write an article explaining this stuff, but I have no time for it now.”””

        Please do it! A nice plot of fermion fermion scattering as calculated from superstrings, another one of boson boson, and their equivalent for point particles. Or just the fermion case if you are in hurry.

    2. @Vincent: Why should anyone think that particles may not have a detectable size? Because the electron’s field is what it is. And there is no edge to it. Take a look at some pictures of a vector field. Then take a look at some pictures of a hurricane. That’s no point particle. The eye might draw your eye, but that’s not what the hurricane is. And there is no edge to it. Take away that intrinsic spin by throwing a cyclone at an anticyclone and all you’ve got is wind. Take away the electron intrinsic spin by throwing a positron at it, and all you’ve got is light. Then look at a wind wave under the surface*. There is no edge to it. It has a wavelength. It has an amplitude. But it doesn’t have a size.

      * http://en.wikipedia.org/wiki/Wind_wave#Science_of_waves

      1. duffieldjohn: If you take a box of a large quantity of steel ball bearings and drop them from a good height on to the ground and you say look there is no size here to the scattered ball bearings simply because there is no easily defined size to the scattered pile have you really ruled out that matter at its most basic level is not granular? At the quantum level we only see the scattered pile because the things that we use to inquire at that level are at similar or bigger sizes than the things we are wanting to look at. I don’t think that an inability to determine an exact size is the same as concluding that some things like photons and electrons don’t have a size even if calculations in a theory don’t need to have a determination of size for the theoretical model to work.

        With the above said I do acknowledge that many phenomena manifest itself to our consciousness by way of waves, yet I think it is wrong thinking some people seem to have that treats all phenomenon as being only due to waves. I think that is wrong because with waves there are wave crests but no bounce back. Bounce back and scattering is better explained, I believe, if we treat some things as intrinsically solid in that they occupy space to the exclusion of other things.

        1. Vincent: those ball bearings don’t let you rule out that matter at its most basic is granular. Electron diffraction does that. And neutron diffraction, et cetera. People say that scattering experiments show that the electron is very very small. But that’s something like crashing two standing waves together so that they pile up and concentrate their energy into a small location and appear pointlike. The electron just isn’t some point-particle thing that’s “got” a field. Field is what it is.

          veeramohan: the photoelectric effect demonstrates the quantum nature of light wherein there’s an h in E=hf and E=hc/λ. There’s frequency and wavelength in those expressions. Look at Susskind’s lecture* 2 minutes 50 seconds in. See how he waves his marker around when he’s talking about angular momentum? That’s like the rotation of the test particles under the water. The analogy is that all ocean waves are the same height, not that they consist of ball bearings.

          * http://www.youtube.com/watch?v=JqNg819PiZY

  26. Dilaton | September 17, 2013 at 10:50 AM | Reply
    That would be a from a scientific point of view pointless wast of time.

    No scientific discussion or debate is pointless or a waste of time as evidenced by the exchange between Woit and Strassler. It often hellps to further clarify things. I couldn’t disagree with you more.

    1. Ok yes,

      Prof. Strassler does a beautiful job in counter and clarifying things , which could help the readers understand what is going on and every reasonable person can then clearly see from these discussions who is right and who is not …

      What I rather meant would be pointless, is trying to convince people who have their own personal opinion ( in contrast to knowledge about the topic) to listen to rational scientific arguments, such people are not reachable.

      But to reach unreachable people by such a discussion is probably not the goal of such a discussion anyway, so maybe I probably understand now better what you meant in the above comment (?) and appreciate it.

      Cheers

  27. I like this very very nice and much needed article!

    Concerning the rubbish

    “Results from the Large Hadron Collider [LHC] have blown string theory out of the water.”

    I think we all know which one of the commenters below always starts to invent and promote such wrong and misleading things intentionally and on purpose … 😀

    So, whereas I really enjoyed reading this post, I will not into the comments futher here for obvious reasons …

  28. String theory is said be some (e.g., Peter Woit) not to be falsifiable, and thus not part of science.

    But people right now are experimentally testing basic questions regarding quantum mechanics (superposition, entanglement, etc.) as well as doing sub-millimeter tests of gravity, and if it turns out that quantum mechanics is ultimately not exactly correct, or gravity disappears on, say, the nanometer scale, then string theory, which is built on quantum mechanics and necessarily includes gravity would be falsified.

    So how again is string theory not falsifiable?

    1. Woit is also giving an answer to the wrong question, in my opinion (though he’s doing so in reaction to the string hype machine, which has promised way, way too much for decades).

      Quantum field theory, as a general framework, wouldn’t have been falsifiable either, if you asked about it in 1950; there are a gazillion quantum field theories, and if simple quantum field theories hadn’t worked for the Standard Model, that wouldn’t falsify quantum field theory, because there are so many examples with so many properties.

      What is easily falsifiable is string theory in particular vacua, or string theory in large classes of vacua. This is analogous to the statement that individual quantum field theories, and certain classes of quantum field theories, are easily falsifiable. But quantum field theory in complete generality?! Very difficult to falsify.

      There are one or two statements that I’m aware of that apply in almost all quantum field theories (e.g. the Froissart-Martin bound on scattering rates.) But those statements were not easily obtained in field theory, and they are not trivial to check experimentally. Similarly, there may be very general statements about string theory that apply in almost all string theory vacua. These also will likely be difficult to obtain theoretically, and impossible, practically, to check in the near- or medium-term.

      1. Yes Matt, my short-hand claim “string theory makes no predictions” is obviously a simplification of a very complicated situation, one that has been exploited for decades by string theorists making bogus claims for predictions. Of course I agree with your headline that the LHC can’t rule out “string theory” (no experiment, practical or impractical can rule it out since it is so ill-defined). The reason some people have this idea though is just that for many years string theorists answered the “no predictions” argument with “we have predictions, for SUSY or extra dimensions at the LHC”.

        In general you do a good job with trying to make accurate statements here and inform the public. Some of what you’re doing in this posting though (eg. claiming the string theory situation is the same as the QFT situation) is just reinforcing dubious hype designed to mislead people, and not up to your usual standards.

        1. Got a little wire-crossed on the replies here; see my reply to a different comment for more details.

          I am trying to split the difference between what I see as your over-statements and the string supporters’ over-statements.

          They were ridiculous to say string theory predicts SUSY and/or extra dimensions at the LHC — and the best string theorists did NOT make that statement, in my experience… and my experience counts for something, because I was sitting right next to them at lunch, day after day.

          But you in turn are not helping the situation by (A) giving people the impression that everyone in leadership positions within high-energy physics is a “string theorist”, including people like Polyakov, Seiberg, Arkani-Hamed, Randall, and the like; and (B) saying that all of these people are working on completely non-predictive ideas that shouldn’t even be considered science. Those are fighting words, Peter — certainly false, and extremely dangerous to the field’s future. You are insulting the intelligence and denigrating the important theoretical contributions of some of the world’s best scientists.

          So personally I’d like to see you tone things down a bit, and give people their due.

          1. The “fighting words” you attribute to me are things I’ve never said and things I don’t believe, and the only one here insulting people is you.

            Yes, there is and has been for quite a while a danger to the field’s future, but I’m not the source of it. The large amount of hype of failed ideas and promotion of multiverse pseudo-science that has been going on for all too long is the problem, not my pointing it out. You’re not helping matters by your attempt here to “split the difference” and claim that the problem with string theory unification is just “impracticality” and the obnoxiousness of a certain blogger.

          2. @Peter Woit: every intelligent reader can clearly see who is reasonable from a scientific point of view in this discussion, who is misbihaving (such as spreading from a scientific point of view misleading and wrong statements), who is attacking / accusing whom, etc … 😉

            So I second Prof. Strassler’s advice to tone and calm things down, everything else does make you look not so good …

            http://blsciblogs.baruch.cuny.edu/fro12bwa/files/2012/12/crocodile-tear.jpg

          3. @Dilaton please stop trolling this intelligent discussion between Dr Woit and Prof Strassler, which the rest of us are learning from, unless you have something informed to add. You’re poisoning the discussion with your abusive comments to Dr Woit 🙁

            People have a right to see both sides of the argument, and Dr Woit and Prof Matt Strasser both provide interesting, if a little conflicting, view points, that the rest of us can look into further.

      2. There’s a distinction we make in Computer Science which I think would help clarify the argument quite a lot: between “decidable” and “semi-decidable”. A semi-decidable property is one which, if it’s true, will eventually turn up, but if it’s not true, you will never know, because it might turn up if you wait a bit longer. A decidable property is one which you will know the answer in a fixed time either way.

        One of Woit’s regular complaints is about people making semi-decidable predictions, such as “super-symmetric particles will show up, but we can’t say at what energy level”. Is this a testable prediction? Is it a scientific prediction? It is certainly qualitatively different to “super-symmetric particles will show up at the energy level of the LHC”.

        If you agree with Popper, semi-decidable predictions are only “scientific” if they are negative.

        1. I would agree with Woit that a statement such as “super-symmetric particles will show up, but we can’t say at what energy level” is not a scientifically useful prediction. Whether you do or don’t call it “scientific” is a semantic issue; it doesn’t matter to me.

          A slightly better, but still very vague prediction, is that “superpartner particles will show up before we reach the mass-energy of the lowest lying black holes (i.e. the real Planck scale, whatever that turns out to be.)” This one WOULD BE a testable prediction, though very vague — but it is probably impossible to test in practice. This would be true in a large class of string vacua, but not all. Is this really not a prediction at all? Here I think we see the issue of principle versus practice; and in my view Woit goes totally overboard, driven by his irrational hatred of string theory and string theorists to make unreasonable statements that these don’t count as predictions.

          Of course “superpartners will show up at 600-700 GeV” would be a nice prediction. Specific theories can predict this. But LHC-scale supersymmetry has many variants, so all we can say is “if supersymmetry solves the naturalness problem some superpartners should lie somewhere below 1000 GeV”. That’s pretty darn vague too… but it IS testable in practice. It is not (as I can tell you from painful experience) easy to falsify completely.

          Similarly: in quantum field theories that aren’t scale-invariant in a non-trivial way, one can predict there are particle states. That’s a very vague statement. First, you have to agree that you’re not in a scale-invariant quantum field theory. Second, nothing tells you what the masses of the particles are — they could be anywhere. Now, is this a scientific prediction? Yes, I think so: it tells you a basic fact about quantum field theories of a certain class. I don’t see the point of trying to argue that this makes quantum field theory as a framework non-predictive. If I’d given you a specific quantum field theory, it would have been perfectly predictive.

          You have to think of string theory as a framework. A string theory IN A PARTICULAR VACUUM is perfectly predictive — though none of the predictions may be testable in practice, because of experimental limitations.

          It is perfectly reasonable to think this way. Take the theory of planetary formation. This theory does not tell you where the planets will form around the sun. Oh My Gosh. What an awful theory. You actually have to input some information — namely, the shape and composition of the initial nebula out of which the sun and planets formed — if you want to predict where the planets ended up. In fact, given that there are 10^500 possible nebulae (or more), there are 10^500 possible solar systems. Does that make the theory non-predictive? Of course not.

          Similarly: in any theory in any scientific context, the theory requires some initial data, without which predictions are limited or impossible; once you specify the initial data (and you have to learn it from somewhere using experiments), you can predict what happens. Just because some initial data have to be learned from outside the theory — in the string context, the relevant data is “which vacuum (or type of vacuum) are we in” — does not make the theory non-scientific or non-predictive. It’s simply more limited than you might like. The string theorists over-sold their theory; Woit is underselling it; don’t listen to either of them, just think carefully and listen to sensible people without an axe to grind.

          1. The problem with your analogy is that the composition of the initial nebula can be predicted using our theory of galaxy formation, and that is testable from other observations. To make the analogy work better, we need a theory of vacua (at very least, a probability distribution) to predict the initial conditions. But that seems unobtainable, because the only “observation” we have is this one universe.

            Your defence of string theory as a foundational technique, analogous to QFT in the standard model and differential geometry in GR, seems pretty solid to me. Unlike Woit, I would never argue against people studying whatever they feel is most likely to generate useful results.

            1. “The problem with your analogy is that the composition of the initial nebula can be predicted using our theory of galaxy formation, and that is testable from other observations.”

              No, I think that’s not the right way to think about it. There’s no practical way to predict the composition of the initial solar nebula; it is determined by the (typically chaotic) magnetohydrodynamics of a gas cloud, whose details cannot be specified by the theory of galaxy formation unless you had all of *its* initial data; and in turn that initial data cannot be specified unless you had the theory of cosmology with all of *its* initial data; and that can’t be specified unless you know which theory of quantum gravity is right and what all of *its* initial data are. You must know the precise initial conditions of the universe and the precise equations if you are to predict the formation of the Milky Way galaxy, the formation of the sun’s protonebula, and the formation of the Earth, Venus, etc near the sun. (Worse, of course, the formation of the planets may depend on other nearby stars’ locations, but let’s set that aside.)

              If you want to predict the precise locations of the sun’s planets, you need precise initial data.

              Even if you want to predict that the sun has any planets at all, you need some initial information.

              Without initial data, you may be able to predict that the typical star has a few planets, and maybe even the average number that it has and their average distances or something like that.

              Similarly, you can expect gradations of things you can predict in other theories, including theories of quantum gravity. Some things are general; some things are specific; the specific require more initial data.

              If you want to predict the precise particles we have in nature, *and string theory is correct* (of course it might not be), then it would appear you need a lot of initial data. Many string theorists were so in love with the beauty of their equations that they used to believe this wouldn’t be a big problem. (I never believed them, and I wasn’t alone.) They either believed that one wouldn’t need initial data or it would be obvious what the initial data were by elegance criteria. But in fact the initial data problem is spectacularly bad — as bad as in trying to predict exactly where the planets are around the sun, or maybe more like trying to predict where all the asteroids are in the asteroid belt.

              A theory of vacua (in any theory, not just string theory) will *not* tell you which vacuum you are in, especially in a cooling universe that goes through phase transitions where accidents can happen. It might, if you are very lucky, give you a reason to think you’re in one class of vacua rather than another. Similarly, a theory of nebulae might give you a reason to expect rocky planets around certain types of stars. But there’s no guarantee it will do that.

              All of this is part of the normal scientific process of learning what a theory can and cannot do for you. People used to try to predict the strength of electromagnetism (about 1/137, see http://profmattstrassler.com/articles-and-posts/particle-physics-basics/the-known-forces-of-nature/the-strength-of-the-known-forces/) from first principles, as though a theory could be found that explains it. But over decades, people learned how quantum field theory works; they realized that physics is determined at short distance in quantum field theory, not at long distance; that 1/137 actually changes with distance; that to predict it you would have to know things about very short distances and combine that information with the specific masses of all the electrically charged particles; and in fact you only get electromagnetism after the Higgs mechanicsm mixes the weak isospin force and hypercharge forces together, so in a sense electromagnetism isn’t even the fundamental thing you should be trying to explain. What’s my point? It turned out that asking “why is the strength of electromagnetism 1/137?” was a bad question. Similarly, it turns out that if string theory is right, asking “why does nature have the specific particles and forces that we find in experiments” might be a bad question. It might be determined by initial data, or by accidents that occur during phase transitions, etc; it might not be possible for string theory to predict this. That doesn’t make the theory non-predictive; it just tells you that this question wasn’t a question the theory could answer. And that’s NORMAL. All theories have questions they can’t answer.

              The only reason there is controversy is

              1) certain string theorists over-hyped the theory, made over-arching, unjustified claims, took over entire theoretical physics groups, and pissed everyone else off — but that’s not a reason to get mad at the theory itself.
              2) there’s no reason to have hope that any measurements that we will make in our lifetimes or those of our children will tell us enough for even a person who understood all the relevant vacua of string theory to guess which one we’re in. And so we’re not, for practical reasons, going to know the initial data, and therefore, for practical reasons, we’re not going to be able to make any precise predictions — and maybe any predictions at all — using string theory. If Woit would limit himself to that statement, I’d agree with him.

          2. Everything you say about the practicality of predicting the initial conditions is true, and as far as I know it might even make probability distributions impossible to calculate, but there’s still the logical, causal relationship. We can consider changing our theory of planetary formation in the light of new knowledge, or our theory of galaxy and nebula formation, or even both, but the relationship between them wouldn’t change: galaxies form dust clouds, dust clouds form solar systems. (Obviously there’s feedback, but that doesn’t change the argument).

            But in the absence of a theory of vacua, how do we decide what is initial? If you accept string theory as true, then the initial data takes a particular form, but other theories take different sets of parameters, making different things initial. If string theory is merely a mathematical tool, then are we justified in assuming that the initial conditions of string theory are initial in any causal (let alone physical) sense?

            1. I think, a bit like Woit, you’re still mistaking issues of principle for issues of practice.

              First of all, this has nothing to do with string theory in particular; this is a problem for any quantum gravity theory in cosmology. So let’s not even use the word string theory here.

              Take any theory X for quantum gravity. It will generically have multiple vacua, maybe not 10^500, but very possibly thousands or millions or 10^10. All you need is elementary or composite spin-zero fields; now you’ve got a problem, because you won’t predict exactly which vacuum you end up in. What is certainly the case is that in a cooling universe, or an inflationary one, where you end up depends on initial data… how did you get the whole thing rolling at the earliest times? Yes, this is a very, very hard problem, because you can’t play around with the universe experimentally, or watch other universes evolve. But this is a practical problem, not one of principle. It is a very serious practical problem. But it is not obviously insoluble. It is possible that a clever enough person with a powerful enough computer or mathematical tools can work backwards from what we observe in the world to figure out which vacuum (or which type of vacuum) we must be in — and then, armed with that initial data, make predictions about other aspects of the world.

              Again, this is not so different from working backwards from what we know about the solar system to figure out what the initial conditions of the pre-solar nebula must have been like; and then, having determined this, make predictions about other aspects of the solar system that we don’t yet know, as a way of testing the theory of planetary formation.

              In this regard, what is so special about string theory? We will have exactly the same discussion about any theory of quantum gravity… unless that theory has one and only one vacuum, which (if one studies quantum field theory, and quantum gravity in general without the specifics of quantum field theory) does not seem very likely. So I honestly just don’t see what all the fuss is about here…

          3. > You have to think of string theory as a framework. A string theory IN A PARTICULAR VACUUM is perfectly predictive — though none of the predictions may be testable in practice, because of experimental limitations.

            I guess many people wonder why we consider “string theory” to be a part of physics (and host it in physics departments) if none of its predictions can be tested practically? After all, the definition of physics is to understand and explain current experiments and make predictions for future experiments. And by “future” I mean foreseeable future, because if we have a theory which (maybe) makes predictions for experiments 200 years from now, why are we studying it today? I personally do not like “falsifiable in principle” argument because not only we can’t even dream of building experiment to explore Planck scale physics, we don’t even know that it is in principle possible.

            And if someone argues that string theory is a useful tool for physics calculations, algebraic geometry is also used in the same calculations, but no one calls it physics.

        2. “Unlike Woit, I would never argue against people studying whatever they feel is most likely to generate useful results.”

          This comment thread is just full of accusations like this that are absurd (another is that I “hate string theorists”. actually, some of my best friends are string theorist. Oddly enough, it’s possible to disagree with people about a scientific issue without hating them). Of course I have never made such an argument. Scientists should study what they believe in. They should also not get completely bent out of shape when someone disagrees with what they believe in.

          The issue I disagree with Matt here about is a complicated one: is the problem with string theory predictivity just one of practice (the calculations needed are too hard, the experiments are too difficult), or one of principle (the theory is too ill-defined to answer the questions it should answer)? I don’t think the position that the string theory “landscape” framework raises problems of principle, not just practicality, is an extreme one, rather it’s quite mainstream, with Matt’s claim that string theory’s problems are just practical ones something that many if not most theorists would disagree with.

          Unfortunately, for some reason it’s impossible to actually have a serious discussion of this issue here, since Matt and some of his anonymous fans feel the way to deal with the issue is to engage in ad hominem attacks and attribute absurd straw man arguments to me, while refusing to deal with the actual arguments I do make.

          1. I’ll ignore the sad wails of “poor me”, and address the issue.

            You are free to think what you want. And you are free to think that all sorts of important people who actually understand the issues (as opposed to smart but uninformed people, who might agree with you, but don’t actually know the details of the issues) agree with you. You are even free to say so, without proof. What you are not free to do, as someone who claims to be scientifically trained, is tell the public things that are factually wrong. And if you say that a particular quantum field theory is predictive, while string theory in a particular vacuum is not predictive, then this is factually wrong. Just plain wrong. Not “not even wrong”. Wrong. It doesn’t matter how many people agree with you on this point. It’s wrong.

            So are you saying this, or not?

          2. I have observed many times that in scientific discussions on your own site (ones involving very knowledgeable people like a certain Matt, probably not Strassler, come to mind), as soon as your point of view has been clearly defeated by rational scientific arguments, instead of acknowledging and admitting this you suddenly declare the discussion as off topic, delete commets of people who disagree with you, etc …

            This is fine, since on your own site you can do what you want after all.

            But from what I observe here, Matt Strassler and others are giving nice and thorough answers to your actual arguments. So comparing the style and moderation of discussions here to what can be observed from your site, it is clear what is more serious and reasonable from a scientific point of view.

            And I am sorry to say, but your irrational hatred of string theory and people working on it is plain visible for everybody who follows the writings on your site and other activities in popular media.

            Whining about “ad hominem attacks upon encountering disagreement with your opinion here does make it not better.

          3. I’m sorry I said that, and I retract it totally. I read both blogs and enjoy them both. I don’t consider myself to be a supporter of either point of view: I simply don’t know enough about it.

          4. Ignoring the usual ad hominem nastiness you seem unable to stop yourself from, to deal with the scientific issue here and make an accurate statement, one needs to first address the following:

            1. What is a prediction?
            This thread started with “Matt” saying that string theory is predictive because it predicts that quantum mechanics is valid, and that’s a testable prediction. You ignored this in responding to him, for presumably the excellent reason that that’s a silly use of the term. Do you agree? What really counts as a prediction? Was SUSY at the LHC a prediction of string theory by your definition of prediction?

            2. What is string theory?
            If we agree on what is a prediction and what isn’t, then we have to agree on what string theory is. I don’t think anyone has a precise answer, with typical answers “some unknown theory with properties X, Y, and Z that in a specific limit has as asymptotic series expansion the perturbative superstring. What is your choice of X, Y, Z? Different choices will give different classes of possible predictions.

            3. What are the vacuum states of string theory?
            Presumably you want the old-style compactifications to count, but remember, those are just wrong: they predict unseen long-range forces (you need to stabilize moduli). You have to stabilize moduli, get the right CC, provide a convincing argument that these are actual vacuum states (remember, you’re not sure quite what the theory is, don’t know when or how those black holes will get created), and deal with the issue that if you can get something consistent that looks like the SM out of this, maybe you’re dealing with such a large class of vacua that you can get anything you want if you collide those two electrons at 10^18 GeV.

            What is the difference between this situation and QFT? That’s pretty simple: no problems 2 and 3. And those problems are not problems of calculations being hard.

            1. Your examples of 2 and 3 are straw men, Peter. There’s nothing there. Problems 2 and 3 afflicted field theory in the 1960s.

              Regarding 2: In the 1960s, it wasn’t clear how to define quantum field theory. Defining it required lattice gauge theory and the Wilsonian renormalization program, as you know… a Nobel prize was won about this. In fact, defining chiral field theories and supersymmetric gauge theories is still complicated and advances were still being made in the 1980s and 1990s. Yet this did not prevent people from developing the tools of field theory and even making contact with experiment, with the positive results that you know as well as I do.

              Regarding 3: In the 1960s, nobody understood the vacua of gauge theory. In fact, there were may confusions about Gribov copies and the theta angle, CP violation and the U(1) problem, and until this was understood, quantum field theory made a wrong prediction for QCD: the eta’ particle should have been light, and it wasn’t.

              And you’ve also left out another problem with field theory in the 1960s; few people thought the theory was even defined until ‘t Hooft proved non-abelian gauge theories, even ones with massive gauge bosons, were renormalizable. And after he proved it (gaining another Nobel prize), everything changed.

              So again, you are making a big deal out of something that is a historical accident. Back in the 1960s, a person could have made the same statements about quantum field theory: that is is non-predictive, that it isn’t defined, that you have no idea how to think about the vacuum… and indeed, I believe some people did make those statements, and I know that some of those people ridiculed their colleagues who were doing field theory.

              Thus, when you say:

              “What is the difference between this situation and QFT? That’s pretty simple: no problems 2 and 3. And those problems are not problems of calculations being hard.”

              you are simply wrong. Back in the 1960s, problems 2 and 3 were present. And why were they present? Because these were problems of calculations being hard!

              So who is to say where things will be with quantum gravity in 2030? What gives you special wisdom?

          5. Matt,
            You’re answering none of my questions about the issue at hand: what is the current state of “string theory predictions”, are they available in principle if we could do hard calculations, or is there a problem of principle?

            As for the historical analogies, sure, back in the sixties there were similar problems of principle (which were problems of calculations being impossible in principle, not just “too hard”) with non-perturbative QFT (some may even still remain). Probably the majority of theorists were skeptical they could be overcome, and they were wrong. It’s possible that I (and the majority of theorists) am now wrong, and 2030 will see some wonderful new insight into string theory that will make predictions possible, in principle or in practice. But that’s not where we are now, and you’re confusing reality with some hopes and dreams which have not shown any signs of working out for the past few decades (quite the opposite, actually…).

            1. 1) You don’t answer any of my questions either. See the previous comment, which you have not answered. Do you claim that a particular quantum field theory allows predictions, whereas string theory in a particular vacuum does not?

              2)”what is the current state of “string theory predictions”, are they available in principle if we could do hard calculations, or is there a problem of principle?”

              I will answer this. Let’s go back above to my discussion of string theory at the very beginning. I stated: there are [at least] two distinct applications of string theory. Application number 1 is to a theory of everything; I am not interested in that and have never written any papers on it. Application number 2 of string theory is as a tool to study other physics, such as quantum field theory.

              A portion — certainly not all, but some — of my research has involved application number 2. How does it work? We pick a particular string vacuum — one which is not maybe interesting to you, and certainly not interesting for Application number 1, but appears to make sense: a string theory in a supersymmetric vacuum, with a negative cosmological constant, which is asymptotically an Anti-de Sitter space. Again: not interesting for Application number 1, but very interesting for Application number 2.

              And in that vacuum, WE CALCULATE SOMETHING. Something definite. We make computations, rigorous and well-defined; often approximate, but most calculations in physics are approximate. The details of string scattering actually matter in that computation. And from that computation, we obtain information about a quantum field theory — which as you have just said yourself doesn’t (any longer) have problems 2 and 3.

              Now, if string theory in a particular vacuum couldn’t make any predictions, how could that possibly work?!

              How could it be that I can learn about a quantum field theory, without problems 2 and 3, by doing a calculation in string theory, if, as you claim, calculations are in principle impossible in string theory?

              And if that were the case, why do you find papers by people like Larry Yaffe (nuclear physics, Washington), Krishna Rajagopal (nuclear physics, MIT), Subir Sachdev (solid state physics, Harvard), Al Mueller (nuclear and particle physics, Columbia), Dam Son (just about everything, Chicago), and Urs Wiedemann (nuclear physics, CERN) that use string theory to gain insights into their own research fields?

              For example, see Wiedemann’s talk at the LHCP conference in Barcelona this year. Application number 2 of string theory comes up already on page 6 (and he refers to a calculation by Yaffe on page 18)
              https://indico.cern.ch/getFile.py/access?contribId=51&sessionId=1&resId=0&materialId=slides&confId=210555

              And here is Sachdev’s writeup on the issue: http://qpt.physics.harvard.edu/c63.pdf

              Or how about this: Polchinski and I calculated, USING STRING THEORY, the so-called BFKL exponent in a supersymmetric gauge theory; and the same exponent was calculated by a famous quantum field theorist, Lev Lipatov, with tour-de-force mathematics USING THE FIELD THEORY; and we got the same answer. How would this be possible if string theory was so ill-defined, so poorly understood, that one cannot make predictions at all, in any of its vacua?

              I could go on and on in this vein. If you force me, I will… although, unlike you, I actually have collaborators about to kill me for spending time fighting an indefensible argument instead of finishing two timely papers.

              Now Peter: are you saying all of this is just accidental? That none of these calculations mean anything at all? Or are you saying that calculations of how things happen in a theory don’t count as predictions?

              1. p.s. it must be really hard for you to see Larry Yaffe working on these things. You and I both know how smart he is, and how connected he is with real physics. Do you really think he’s not doing science anymore?

          6. Matt,
            You really have the most bizarre ideas about what my views are. Starting with the weird p.s., some background: I spent my graduate student years trying to learn everything I could about ideas for doing non-perturbative calculations in QCD (and about geometry and topology of gauge fields). One thing I tried hard to learn was work of Polyakov, Migdal and others on string theory as a way to do QCD. Finding some kind of “string dual” seemed to me then and still seems to me the most promising way to really understand QCD. I’ve no problem at all with people using string theory to try and solve strong-coupling QFT problems, problems in algebraic geometry, or anything else, quite the opposite. Much of this is very good work (by the way, quite a few years ago, even though I wasn’t often going to talks over in physics, I went to your seminar here, enjoyed it a lot). My only complaints about this huge area of research just have to do with some people using it to mislead the public into thinking this kind of progress shows string theory unification is getting somewhere.

            About Larry Yaffe: he was quite helpful to me when I was a student, I think he’s an excellent physicist, don’t know much about his current work, but don’t doubt that it’s of high quality. The idea that I might think it’s a bad thing that his work has some connection to string theory is just absurd. I just took a look again at something he wrote for posting on my blog nearly 10 years ago, soon after I started it, and it’s every bit as wise as I remember it, see
            http://www.math.columbia.edu/~woit/wordpress/?p=69

            String “vacua” designed to provide string duals to QCD are just completely different beasts than the ones that have to do with string theory unification. There’s not really a “predictivity” issue there, since it is QCD that one is supposed to be confronting with experiment, string theory is an approximation method. You’re not testing a theory, you’re testing an approximation scheme, which is a somewhat different issue (and yes, I’ve complained in my blog about attempts to muddy the waters about this difference).

            My claims about what string theory vacua I was referring to (the ones that are supposed to give a unified theory) were made explicitly above, and I tried to point out one that needs to make clear what purported vacua one is referring to and what one means by “string theory”. I’ve never argued that there is no such thing as a sensible string theory calculation. About QFT in general, I don’t even know what you’re trying to get at. There are lots of kinds of QFTs, and in each different case we know different amounts about their vacuum structure.

            Finally, if I were you, I’d think long and hard about what conclusions your colleagues might draw about your insistence on including personal attacks (“unlike you, I actually have collaborators”) in your scientific arguments. “Dilaton” likes that sort of thing, most people detest it.

            1. “My only complaints about this huge area of research just have to do with some people using it to mislead the public into thinking this kind of progress shows string theory unification is getting somewhere.”

              Really, that is your only complaint? If so, why did you complain about my post above in the first place? Did I, through my post, mislead the public in this way? If you really think so, I’m sorry, we’ll just have to agree to disagree. I said absolutely nothing that suggested that at all. Especially since I said we aren’t even remotely close to being able to test anything related to it.

              “String “vacua” designed to provide string duals to QCD are just completely different beasts than the ones that have to do with string theory unification. There’s not really a “predictivity” issue there, since it is QCD that one is supposed to be confronting with experiment, string theory is an approximation method. You’re not testing a theory, you’re testing an approximation scheme…”

              Huh? I’m sorry, I don’t agree with this at all. I’m not talking about string duals to QCD, which are approximate. I’m talking about string duals to other field theories (not QCD) that are believed to be EXACT. No approximation scheme. As you know, there’s considerable evidence for this duality. Or yet other theories, for which we have no data at present, and the approximation scheme is all we’ve got.

              Now I hope you’re not going to tell me that making a “prediction” for the spectrum or scattering behavior of a field theory that we don’t find in the real world doesn’t count as a true prediction. We do not have a gluons-only (Yang-Mills) theory in the real world. But calculations using lattice gauge theory of the spectrum of hadrons (“glueballs”) in such a theory are most definitely a prediction for what you would find if you lived in such a world. And of course, those predictions might, in fact, be relevant in the real world; if there is an as-yet undiscovered force with massless gluon-like particles but no quark-like particles, the spectrum of particles that our descendants will uncover in their experiments are indeed computed by that lattice gauge theory. The prediction stands; it may someday be verified in data.

              Now, Klebanov and I found a string theory method to compute the spectrum of a particular quantum field theory (“the duality cascade”). This is a result that is correct up to 1/N and 1/(t’Hooft coupling) corrections. If, in fact, this field theory turns up in nature somewhere, we have predicted its spectrum. Period. This is not about approximating QCD. It’s about making a prediction for a quantum field theory that very well could exist, though it might not.

              I hope you don’t try to argue that this doesn’t count as a prediction. Because if you do, every calculation that anyone has ever made in any theory other than the Standard Model itself is not a prediction — which means that every quantum field theory except the one in the real world is a non-predictive theory and isn’t science.

              “My claims about what string theory vacua I was referring to (the ones that are supposed to give a unified theory) were made explicitly above, and I tried to point out one that needs to make clear what purported vacua one is referring to and what one means by “string theory”.”

              Again, what does this have to do with your original objection to my post? I did precisely these things. I said: “(taking string theory in its most vanilla form)”. That’s pretty darn clear that it’s not generic; if you know what I’m talking about already, it’s clear what I mean, and if you don’t, explaining it wouldn’t help. Obviously I agree that one has to be clear about this — that’s why I made the statement in the first place. I didn’t go into details because they are very technical. But I did exactly what you say you wanted me to do. So why did you complain?

              Back to some of your comments that I let slide:

              You earlier said “You’re not helping matters by your attempt here to “split the difference” and claim that the problem with string theory unification is just “impracticality” and the obnoxiousness of a certain blogger.” Of course I did not make such a claim. It’s a straw man. I did exactly what I should do: I explained that string theory makes predictions in *particular* contexts. Your book and your blog give the impression (and I know this, because of frequent questions from commenters) that string theory is just philosophy and navel-gazing. That, of course, is false.

              You also asked “Was SUSY at the LHC a prediction of string theory by your definition of prediction?” I ignored this question because it is another straw man; you know perfectly well I don’t think so, because I said so *explicitly* in the post above.

              And you also said “my short-hand claim “string theory makes no predictions” is obviously a simplification of a very complicated situation, one that has been exploited for decades by string theorists making bogus claims for predictions”. Well, your short-hand claim is extraordinarily irresponsible (and demonstrably false, see above). The same claim has been exploited in recent years by people who want to prevent me from raising money for my research… and they succeeded, twice. You already know of one case; you can probably guess a second, if you think about it. I certainly don’t know how much your own efforts are to blame, so I won’t blame you, but this is what you’ve aligned yourself with.

              In any case, I can’t for the life of me figure out what your original complaint about my post was about. If you’re really only complaining about string unification and the Brian Greenes of the world, then stop telling the public that string theory isn’t science and that it can’t make any predictions. My own string theory work, and that of Yaffe and Sachdev and Son and the rest, is most definitely science, and I resent the implication that is inherent in your short-hand claim, which is (in my experience) the main thing that the public and politicians and other scientists actually remember from your blog. Your words have consequences (far more than mine, given how large your readership is) and you ought to take some responsibility for them.

          7. Telling you the unconvenient truth and facts that you probably dont appreciate to hear, are not “personal attacks”.

            I guess many intelligent and reasonable people (including other high energy physicists) will appreciate the time and effort Matt Strassler (and Joseph Conlon to a less intense degree too) devote to explain and clarify things and rectify the misleading mess often done in popular media (sometimes even on purpose) and not so reasonable (to say the least …) discussions at other places in the internet.

            I was first very sceptical about the usefulness of such a discussion, but now I have read some very interesting things in some comments. In particular the promised (slightly technical I hope?) article about difference between stringy and “ordinary” QFT scattering amplitudes I would appreciate a lot … 🙂

          8. @Matt Strassler
            “… The same claim has been exploited in recent years by people who want to prevent me from raising money for my research… and they succeeded, twice.”

            This is horrible, the irresposnsible and criminel people doing damage to science and preventing good physicists like you from obtaining the money needed to do their job like this (by inventing and promoting dishonest and wrong claims in the public with exactly the intention to damage science and/or make themself known) should be severely punished and efficiently stopped !!!

            Reading about this gives me a hard time to stay at least roughly polite … :-/

            I hope that at least some of the people misguided by darker places in the internet or from reading misleading books etc will come here to learn about how science really works and is done and to get rid of their prejudices and wrong impressions about high energy and other fundamental physics …

          9. Your posting here goes on at length about string theory unification “predictions” of exactly the sort I find misleading and problematic. I complained, you (partially) addressed the complaint in your addendum to the posting. The issues you are bringing up about tests of string theory as a tool to study QFTs are just a different topic, not one I was complaining about. My only complaint about claims of such tests is when they are made in a way designed to obscure the failure of string theory unification (“string theory is so testable, at RHIC”). People doing this are sometimes those heavily invested in string theory unification, trying to evade the consequences of the failure of this program. I know this is not your story.

            On the other hand, some people doing more sensible things with string theory think its a good idea to promote a connection with string unification, figuring that the good PR of that idea will help them. I honestly have no idea if that’s your story. In any case I’m not going to apologize for countering the intense PR behind a bad research program with accurate information about the true state of affairs.

            To whatever extent string theory has been getting bad press in the last decades, I suppose I may have had some sort of influence, but I think much more influential has been people seeing the reaction of string theorists to criticism, together with the serious lack of progress in the field in recent years. As for how influential my views are, my endlessly repeated arguments that theoretical physics needs more mathematical physics in non-string theory directions have never gone anywhere, with doing non-string theory mathematical physics about the best way to make oneself unemployable in any US physics department.

            I can assure you that extremely few people have ever consulted me for advice on how to spend money on theoretical physcs, and I’ve certainly never in any way criticized you or the kind of research you’re talking about in that context. I can only generally guess what fundraising efforts you’re referring to, and from the little I know about ones pretty far in the past I can’t see how I was your problem. I have no idea what you’ve been up to along these lines in recent years, happy to discuss privately, doesn’t seem like a topic for a blog comment section. Problems with getting money for research related to string dualities I would think would have little to do with me, a lot to do with the fact that the Maldacena paper now has about 10,000 citations, so you’re competing with everyone else in the world for support in that area, in a bad environment for US government funding. But, I’m well aware that I have about zero idea of what you’re actually referring to here.

            1. “Your posting here goes on at length about string theory unification “predictions” of exactly the sort I find misleading and problematic. I complained, you (partially) addressed the complaint in your addendum to the posting.”

              `At length’ = 2 paragraphs. Much ado about not very much. All I did was make true statements that counter the impression, held by many of your followers, that one can’t calculate anything at all in string theory.

              Is it not “misleading and problematic” for you to say “string theory makes no predictions” without a proper addendum to your own comments? Do you think such sound-bites do not damage the reputation of serious scientists whose work in string theory, like mine, has nothing to do with Application Number 1 (`string unification’)?

              I certainly have no intention of identifying myself with the theory-of-everything hype machine. I never have done so at any point in my career, and indeed this is clear from the fact that this website, which has lots of particle physics webpages, and has pages that explain the basics of extra dimensions and supersymmetry (with scarcely a mention of string theory, and under the heading “some speculative ideas for the LHC”), has no string theory section at all. [Someday I may make one, but it is low priority, and in any case it won’t say much you’ll disagree with.] My position is also clear from a video (unfortunately poor quality) of a talk I gave about string theory from 2006 (“Beyond the Hype: The Weird World of String Theory”), recently posted at the bottom of http://profmattstrassler.com/videoclips/. It is much more even-handed about string theory than any lecture that I expect you would give, and I believe it gives a fair picture of the history.

              But back to the science. I refuse to concede the original point. If you tell me that string theory for quantum gravity and beyond is a non-predictive theory, I will remind you that in certain vacua, string theory is actually more predictive — well understood, and actually easier for calculations — than certain field theories are. And I challenge you to prove this statement false. (If you don’t know what I’m talking about, try to predict the hadron spectrum and scattering amplitudes of a typical strongly-interacting chiral gauge theory, and tell me that’s easier than predicting the spectrum and scattering amplitudes on certain simple compactifications of supersymmetric string theory backgrounds.) The fact that these string vacua have nothing to do with the real world (and perhaps these field theories don’t either, though we don’t know that for sure) is irrelevant to the basic point: your blanket statement that “string theory makes no predictions”, to be contrasted with quantum field theory in a completely general way, is a confusing, damaging, and arguably slanderous remark. If you want people to think you’re being fair-minded, then don’t make obviously unfair statements.

              To put this another way: it would be nice if you could come up with a way of fighting your continuing battle against the string hype machine (which, judging from last week’s conference, http://profmattstrassler.com/2013/09/16/a-quantum-gravity-cosmology-conference/ , barely exists anymore anyway) without injuring innocent people.

          10. Matt,
            You’re making it quite clear that you’re devoted to doing exactly what I find problematic: misleading and confusing people about the status of string unification by refusing to distinguish between two completely different technical issues.

            Your comments have also been rather eye-opening in explaining the source of your bizarre attacks on me. It sounds like you tried to exploit this technical confusion as part of an attempt to get wealthy people to give you millions of dollars, and when this didn’t work out, you hold me personally responsible. In your mind, you’re just an innocent party, embittered by the injury you suffered in the string wars of not getting millions of dollars. I have no idea what the actual story behind this is, but, no, I’m not sorry at all if my attempts to make clear the problems with string theory unification caused trouble with what you were trying to do.

              1. While Peter does not have the details straight, he is correct about millions.

                If you want to run an international-quality Large Hadron Collider-related theory program, you need something on the order of four postdocs and eight students for four faculty. Otherwise you’ll never keep up with the experiments. The cost of four postdocs and eight students, counting salaries, tuition, benefits and the cut that the university takes, is over $700k per year. So it’s not millions per faculty member, but it certainly is millions for a group of faculty members. Of course government contributes some fraction of this money, but typically $100k per faculty member, and dropping.

                Meanwhile, if you want to run a corresponding program that also includes experimentalists — and one cannot do top quality LHC work if one is not working closely with experimenters — then the cost increases significantly. Of course, government funding increases too — but not enough to keep up with the costs.

                So yes: to run a world-class LHC program requires, in my opinion, that a group of 4 theory faculty and 4-6 experimental faculty raise millions of dollars per year. Some of that comes from the government.

            1. Thank you, Peter. Case closed. Not once, in this entire discussion, have you addressed even *one* of the scientific points I raised. Go back and look through your comments. Not even once.

              And as for your statement:

              “You’re making it quite clear that you’re devoted to doing exactly what I find problematic: misleading and confusing people about the status of string unification by refusing to distinguish between two completely different technical issues.”

              This is an absurd accusation, as anyone who knows my scientific work and my scientific reputation would know. You’re pigeon-holing me, painting me with same simplistic brush you always use. Go watch my video.

          11. I’m completely mystified about why I’m supposed to have had something to do with non-funding of an LHC theory program. You make the point clearly in this posting that the LHC has zero to do with string theory, so why in the world would someone refuse to fund an LHC theory program because I’d turned them against string theory? One would think quite the opposite, that someone looking for HEP theory to fund who had been turned against string theory would find LHC phenomenology the perfect fit.

    2. Wow, a truly long and lengthy discussion. This debate is very helpful and healthy.

      By reading Matt’s original post, he was very fair-handed. By using the hammer analogy, he hinted that the string theory (perhaps, the M-theory types) might not be a viable physics theory but is still good tool for doing many other works, as good as a hammer.

      Then, the debate on the *Predictive-ness* issue made him (Matt) as a string theory strong supporter. This is not what I got from reading his original post.

      For the string theory debate (the M-theory types and the likes), my view is very simple. Any theory beyond Standard Model must make contacts to known physics. The M- or F-string theory has failed on this simple criterion. This failure goes way beyond the *semi-decidable*, the answer is at just beyond the next corner. No, the predictions (even zillions) beyond the next corner will not help one bit on their failure of not being able to make contacts to the known physics.

      With the hammer analogy, I must agree with Matt here.

  29. Matt,
    The problem is your bold-faced “string theory certainly makes predictions”, when, in the modern interpretation of what “string theory” means (M-theory, whatever that is ) only a specific subclass of vacua make predictions. You characterize this subclass of vacua as “large”, but that’s not at all clear, one could just as well argue that they’re probably of measure zero in the space of vacua.

    The argument is not that “string theory” is a useless exercise, it’s that “string theory” predicts nothing at all about particle physics (in the conventional sense of falsifiable predictions, not in the unconventional sense of “if we were to see X it would be evidence for string theory”).

    As for claims that the situation with string theory is the same as QFT, I’ve wasted too much of my life on that obfuscatory debating point. You’re well aware of the huge difference in testabililty between QFTs that have to do with the real world and M-theory.

    1. Peter: I understand your first point, but I’m sorry, by the same logic, the statement “field theory certainly makes predictions” is something an equally stubborn blogger could disagree with. As for whether the space of perturbative string theories is of measure zero; we have no evidence supporting your statement, especially since many non-perturbative string theories that we can study have perturbative subsectors where standard string calculations would apply. All that matters is whether the particles that we know today are perturbatively interacting. In any case, so what? We only care what nature is actually doing. Who cares about the measure on everything else? Do experiments, and see what happens. Then compare with theory.

      I agree that string theory predicts nothing about particle physics. So what? Quantum field theory in general also predicts nothing about particle physics. It is only once I specify a gauge group, matter content, and non-gauge interactions that quantum field theory predicts anything — and even then, it’s hard for a physicist to extract those predictions if there are non-perturbative effects that are hard to calculate, and there can be many surprises. Similarly, only string theory in a particular vacuum will predict anything specific about particle physics; also there are some general things that are probably true in certain classes of vacua. But clearly we will know nothing from string theory on purely theoretical grounds; just as we need experiment to tell us which quantum field theory to work with, we will need experiment to tell us which string vacuum to work in — and we’re not likely to get that type of experiment anytime in the near future. All in all, you and I agree the theory can’t do everything one might like it to do… but that’s true of every theory we’ve ever studied, so what’s the big deal here?

      And as for the third point — I would advise you to go ahead and keep wasting your life on it… right now, you’re wasting your life on something even less useful, trying to convince the world that God has spoken to you and told you how the history of science will proceed. (Talk about wasting your life… how about DOING some science?!) String theory would be just as testable as quantum field theory if we had colliders at the Planck scale. You are confusing a point of principle with a practical problem, in my opinion. I remind you just how long it took to check that quantum field theory explains hadrons… it took seven decades after the discovery of the proton and more than two after the discovery of the proton’s size, EVEN THOUGH experiments were soon to access distance scales corresponding to a proton’s size. Most people did not believe that quantum field theory would succeed. I personally am quite sure that string theory won’t solve the problems that I would like to see solved in my lifetime, but that’s because it’s impractical, not because it’s impossible in principle. And the same is probably going to be true for any candidate theory of quantum gravity.

      Clearly I’m no big supporter of string theory in “Application Number 1” (though I’ve found Application Number 2 very useful.) But I am a big supporter of setting the record straight. I don’t find your methods tasteful.

    2. I work in string phenomenology. Everything done there, everything written there, everything proposed is falsifiable in the conventional sense. Go to high enough energies, and you will find:

      * very rapid growth in the number of states with energy, that can be associated to e..g oscillation modes of higher dimensional objects
      * the appearance of many new degrees of freedom: in geometric compactifications these are associated to extra dimensions, in non-geometric models these are contributing to the central charge.
      * exponential softness of scattering amplitudes at high energy
      * organisation of the new massive particles into supersymmetry
      …..

      Much of the physics of this (for example softness of high energy scattering) follows simply from the presence of extended objects, as the entire string/brane/etc can no longer scatter coherently.

      The difficulty is always to go to LOW (ie TeV) energies, and pick the right observables where short-distance physics has interesting things to say about long distance physics. As Matt has pointed out, this is a practical issue and not a conceptual one.

      If by ‘string theory’, you mean ‘the theory of relativistic quantum mechanical strings’ or ‘the theory that says particles are strings/extended objects’ or ‘the theory with extra dimensions’ or ‘the thing that people who call themselves string theorists and try and connect string theory to the Standard Model call string theory’, you are just wrong.

      You are playing a word game where you use ‘string theory’ in such a nebulous fashion that it doesn’t mean anything. According to you the problem with fundamental physics is that it is dominated by string theory, and string theory makes no predictions, but everything that is actually done in the subject has predictions built in.

      Your argument boils down to an assertion that if there is anything, anywhere that has not been calculated, you cannot say anything. Most people would say that if there are certain features of a theory that have been found always and everywhere after forty years of detailed study in many regimes (including the strong coupling regime), it is reasonable to call these features predictions.

      1. I don’t really disagree in the general sense. But I do think I should point out that when you say

        “The difficulty is always to go to LOW (ie TeV) energies, and pick the right observables where short-distance physics has interesting things to say about long distance physics. As Matt has pointed out, this is a practical issue and not a conceptual one.”

        this is potentially an extreme difficulty. Possibly an insoluble one, for the near or even far future. It is an extraordinary claim, not entirely plausible, that studying physics theoretically at 10^18 GeV can be used to make useful predictions at 1000 GeV. Of course it may be true in some string vacua, but it is surely not true in general.

        It could very well be that non-supersymmetric non-perturbative dynamics, at or below the Planck scale — dynamics that we do not understand today and that we may not be able to understand without major conceptual and mathematical breakthroughs — stands like a wall between what we can calculate using the methods that you use in string phenomenology and what we can hope to measure.

        However, even so, this remains a problem of practice — an extreme problem, based on an inability to calculate — rather than a problem of principle.

        What I learned yesterday is that Woit considers having to specify the vacuum of string theory as an input, rather than as a prediction of the theory, makes the theory non-predictive in principle. This seems very odd to me; all previous scientific theories have required initial data from outside the theory before you can use them to make predictions. By Woit’s logic, the theory of general relativity is non-predictive because you have to specify an initial space-time metric.

        1. “this is potentially an extreme difficulty. Possibly an insoluble one, for the near or even far future. It is an extraordinary claim, not entirely plausible, that studying physics theoretically at 10^18 GeV can be used to make useful predictions at 1000 GeV. Of course it may be true in some string vacua, but it is surely not true in general.”

          The claim isn’t intrinsically any more extraordinary than the claim that by studying physics at E >> 1 TeV, you can make useful predictions for physics at distance scales of (10^(-33) eV)^-1 – but this latter claim is what actually happens in inflationary cosmology, with exquisite and incredibly accurate precision.

          The same is true in axion physics – theoretical models that are defined at f_a > 10^11 GeV can and do make useful predictions for physics at E <~ 10^-3 eV, and are probably the answer for the low-energy problem for why the neutron has no electric dipole moment.

          So a large logarithmic difference in energy scale is no intrinsic barrier. What is more the issue is ensuring that the physics generated at the high scale is protected down to the low scale, which is indeed the case if the high-scale physics has shift symmetries (e.g. axions) or only non-renormalisable couplings (e.g. moduli).

          I would say that low-energy physics dominated by renormalisable operators is insensitive to Planck scale physics, physics where non-renormalisable operators matter is sensitive to Planck-scale physics.

          A few examples (there are plenty more):

          Example: string models generally have moduli, moduli masses are generally comparable to the gravitino mass, but light moduli (m <~ 1 TeV) cause major cosmological problems. So supersymmetric models with light gravitini (i.e. gauge mediation) are disfavoured.

          Example: as above, moduli are cosmologically longlived and cause low reheat temperatures. So low-energy supersymmetry – which is then linked to light moduli and low reheat temperatures – is not compatible with thermal leptogenesis which requires a high reheat temperature.

          Example: string compactifications generally have additional massless U(1)s / axions / WISPs. These are hard to see, but this is because they are weakly interacting, not because they are heavy. There is an active experimental program searching for such particles, see talks by Andreas Ringwald or Joerg Jaeckel.

          I'm not making any claim that there are any unique predictions from all of string theory for low-scale physics (the answer is obviously no).

          The question I am more interested in is: is studying Planck-scale physics useful for suggesting search strategies, and determining experimental programs and priorities at accessible and fundable facilities? To which the answer is yes.

          1. “The question I am more interested in is: is studying Planck-scale physics useful for suggesting search strategies, and determining experimental programs and priorities at accessible and fundable facilities? To which the answer is yes.”

            With this statement I whole-heartedly agree. The wording I used did overstate the point about going from the Planck scale (10^18 GeV) to the weak scale (1000 GeV), and you were right to call me on it. However,

            1) It is far from obvious that studying supersymmetric string theory in particular (rather than string theory with strong supersymmetry breaking, or non-perturbative quantum field theory, or other less theory-specific approaches to quantum gravity) is the best way to carry out this program.

            2) The examples you mentioned (moduli, inflation, axions) and their effects are clearly not unique to string theory; the latter two will likely appear in any quantum field theory coupled to gravity and in any quantum gravity theory, and the former will likely appear in any supersymmetric version thereof.

            In short: “Yes, but what does this really have to do with string theory per se?”

    3. Matt was not misleading in his original post or the amended post addressing Peter’s complaint unless he is wrong that given a particular class of vacua and an understanding of how string theory maps to electrons, you will always be able to say something about the scattering of electrons using that string theory. In all this endless discussion, Peter has never said Matt was wrong about that, let alone explain why he would be wrong about that. Peter has even agreed in this discussion that given a particular vacua and string theory, you can calculate results and those results are useful even though the vacua may not represent any real vacua. It seems clear then that Peter agrees that if you could somehow know the real vacua and how string theory maps to objects, then String theory could then be used to make predictions about electron scattering just as Matt states in his article.

      Peter seems to just be saying it is theoretically impossible to ever know how string theory maps to particle physics and what vacua to use. But there would seem to be know way for Peter to know that and Matt has clearly stated in the article that as of now it’s true that we cannot even imagine how we could do experiments that would help us know what vacua to use.

      Towards the end of this long discussion though, Matt’s comments can be misleading if read out of context from the article and all that was stated before. Taken out of context, it can seem as though Matt is saying that string theory makes predictions simply because he is able to make useful calculations using it even though the string theory is not meant to directly represent what is being studied and is just used as a computational technique like calculus or something. Taken in context though, Matt is clearly using this demonstrated ability to do calculations given a specific string theory and vacua to refute the idea that he is wrong about it being theoretically possible to predict electron scattering using a particular string theory and vacua so that it could be tested against experiment although in practice none of this can be done within the foreseeable future. Matt is giving specific examples of being able to do calculations with String theory just to show that if we know the right initials conditions, we can make calculations with String theory, and those results would obviously be predictions that could be tested against experiments if we had any idea how to carry out such incredibly high energy experiments.

  30. “The overall scattering rate increases with energy.”
    Is there a unitarity bound in ST, so eventually cross sections have to come down?

    1. Eventually you start making black holes, but that’s true in any quantum gravity theory, not just string theory. Once you make black holes, unitarity is presuambly restored… but we don’t know how (see yesterday’s post).

      1. Thanks for clarification: Do interactions other than gravity also have cross sections increasing with energy at these energies? Or are there stronger unitarity bounds from SM? Or perhaps, at that energy ST does not distinguish between different interactions.

        1. No, gravity is special among the known forces. This is because gravity is a force that responds to energy; the larger is the energy, the stronger is the gravitational force at a fixed distance. By contrast, electric forces respond to charge. A high-energy electron has the same charge as a low-energy one, so the electromagnetic force does not becomes stronger at a fixed distance. (This is also correlated with gravity being due to a spin-two gravitational field, while electric forces come from a spin-one electric field.)

  31. You’re claiming that “string theory” predicts the form of the electron-electron scattering amplitude at 10^18 GeV. This assumes the late 1980s, pre-M-theory picture of perturbative string theory on a 6d compactification. If M-theory and non-perturbative effects are important you don’t know this amplitude (for instance, what’s the amplitude for black hole production?). At a recent talk, Arkani-Hamed made a similar point, saying that experiments at the string scale “wouldn’t help you at all”, because the string landscape possibilities are so complex.

    Also, what is the “hard mathematical problem” whose solution would completely explain the black hole evaporation paradoxes of the “firewall” debate?

    1. Peter – let’s leave the firewall question to yesterday’s post, and talk about it another time. I can’t do both sets of questions at once.

      Regarding strings and scattering: if you read my post carefully you’ll see I was fairly precise, though certainly not complete. (I only have finite time and the post doesn’t have space for every caveat, especially ones like M theory, nonperturbative effects and so on that would require pages and pages of detailed discussion.)

      What I said was: “if particles are tiny strings, then…” That certainly isn’t true in every vacuum of the theory. But if you saw this type of behavior for all types of particles, it would give you some indication that particles are stringy in some way. That *is* a prediction of the theory in this very large subclass of vacua.

      I did this because some people have the impression (thanks in part to you, I believe) that you can’t calculate anything in string theory at all under any circumstances… that it’s all talk. That’s of course not true.

      The problem I have with your perspective (and with your quote of Arkani-Hamed, which sounds like an off-the-cuff remark, not one made in a careful discussion) is that you could say the same thing about quantum field theory. For decades people couldn’t calculate what they needed to in quantum field theory; there are many non-perturbative effects, computers weren’t powerful enough to help, … and indeed, even today many quantum field theories can’t be solved well enough to make predictions. Does that make quantum field theory a useless exercise? No; despite the fact that quantum field theory does not predict that in all cases particles will exhibit perturbative scattering, the fact that electron scattering is described by a *particular* perturbative quantum field theory gives us confidence that we can use it.

      So – if I found that Planck energy scattering of all types of particles gave me exponential falloff at large angles and Regge behavior at small angles, I would conclude that string theory in a 1980s-style perturbative vacuum describes these particles well — that the particles are tiny strings. If I don’t find this, I have more work to do to explain what I find. Think about the history of hadrons, and how strings worked well for hadron scattering at first, then were discarded when they failed to describe high-energy hadron scattering. Was string theory useless there? No – it helped people think about flux tubes. But in the end it was quantum field theory (which many people had declared dead, you may recall – you would probably have been among them) which explained hadrons.

      Similarly, if electron-electron scattering didn’t agree with perturbative field theory back 50 years ago, what would I have concluded? (1) electrons are not described by perturbative quantum field theory. (2) there is insufficient theoretical understanding of quantum field theory to say whether a non-perturbative quantum field theory describes electrons or not.

      Anyway, I don’t see the point of your all-or-nothing approach. The history of science does not proceed along a straight path; it proceeds along the braided channels of a river delta.

      I would not conclude: quantum field theory is a pointless exercise just because I can’t make predictions that work in every single quantum field theory.

  32. Your assumption is application 2 speaks as if they exist? Which is very confusing as they may exist but more likely not. If Plank was still alive he may be able to persuade you that they do not. Also I think there is a case to reinvent the meaning of dimension as this too is ambiguous.

  33. Newton & Einstein produced a good equations culminating in the form of STR & GTR. But this only describes and predicts what gravity does it does not explain what causes gravity and comes absolutely nowhere close. In the same breadth I would have to say the same thing about string theory. There is something fundamentally missing in all this classical and modernistic physics. All stemming from Mr Euclid 300 years BC.

    I think P Russkind with P Hawking refs to a Complimentary Function which refers to how you measure and observe things. i.e. depending upon what your measuring or observing will have an effect upon the other.

    This style of physics is almost insatiably crazy – a series of Russian dolls one inside the other to infinity and never discover what is inside the last doll as you will never reach it.

    Obviously we have gained a huge amount of data and insight into how nature ticks but despite the collection of all this information we are nowhere closer to having a clear understanding on the everday simple things which we all take for granted. Being lead astray by this theory and that theory may be popular but from my point of view only causes ongoing confusion.

  34. I’m sure you don’t have time for it, but it would be entertaining to stage a debate between you and some of the popular anti-string theory bloggers. Thank you for the very clear explanation.

    1. That would be a from a scientific point of view pointless wast of time.

      We have seen already way to much of such ugly flame wars, which are only interesting to kibitzers who are neither competent nor interested in the science involved, but who enjoy observing violent debates independent from the topic in question.

      Such “discussions” do science no good, on the contrary they are harmeful and it is exactly the anti-string bloggers who vigorously promote such completely wrong nonsensical statements as Prof. Strassler demounts in this article …

Leave a Reply

Search

Buy The Book

A decay of a Higgs boson, as reconstructed by the CMS experiment at the LHC

Related

Recently, the first completed search for what is nowadays known as SUEP — a Soft-Unclustered-Energy Pattern, in which large numbers of low-energy particles explode outward

POSTED BY Matt Strassler

POSTED BY Matt Strassler

ON 03/15/2024

About a month ago, there was a lot of noise, discussion and controversy concerning CERN‘s proposal to build a giant new tunnel and put powerful

POSTED BY Matt Strassler

POSTED BY Matt Strassler

ON 03/08/2024